Business Law Exam 3 Sample Questions

Ace your homework & exams now with Quizwiz!

Which of the following is a true illusory promise?

"I promise to mow your lawn on such occasions as I designate."

Parr is a CPA licensed to practice in State A. Parr entered into a contract with Jet, Inc. to perform an audit in State B for $50,000 (including expenses). After Parr had satisfactorily performed the audit, Jet discovered that Parr had violated State B's licensing statute by failing to obtain a CPA license in State B. Parr incurred $10,000 in expenses in connection with the audit. jet refuses to pay any fee to Parr, arguing that it could have engaged a local CPA licensed in State B to perform the same services for $35,000 (including expenses). If Parr sues Jet based on breach of contract, Parr will be entitled to recover a maximum of

$0

Water Works had a long-standing policy of offering employees $100 for suggestions actually used. Due to inflation and a decline in the level of quality of suggestions received, Water Works decided to increase the award of $500. Several suggestions were under consideration at that time. Two days prior to the public announcement of the increase to $500, a suggestion by Farber was accepted and put into use. Farber is seeking to collect $500. Farber is entitled to

$100 in accordance with the original offer.

Gus Parker owned a race horse that had not made a good showing in the last 10 races. Disgusted with the horse, Parker stated that he would sell the horse for $25.00. Sam Hood immediately said that he would accept the horse for $25.00. Parker agreed to accept $25.00 after the afternoon's race in which Parker was obligated to participate. The horse won the race with a prize money of $5,000,000, and Parker decided that he did not really want to sell the horse.

$25.00 constitutes sufficient consideration.

Mary agrees to sew Georgia's prom dress for $50 plus costs. Georgia decides that she wants ruffles around the neck and calls Mary who says it will now cost $60. When Mary finishes the dress (with ruffles), Georgia must pay:

$60, since the modified agreement is supported by additional consideration.

Which of the following would most probably be considered a valid offer?

$75 reward for return of lost ring to Arthur Adams, 350 Main Street, Peoria, IL.

A female attains the age of majority at what age?

18

In Florida, courts have ruled that in the sale of a business the following is presumed to be reasonable time for agreements not to compete.

6 months

The UCC provides that a merchant is bound to keep a written offer open for a stated period but no longer than:

90 days

Which of the following normally is an acceptance?

A "grumbling acceptance"

Which of the following statements is/are true?

A business may require, though it may request, a parent to sign a contract between the business and a minor as a precondition for a sale to a minor.; A parent who cosigns a loan with his or her minor child becomes obligated under the loan contract.; As a general rule, parents are not liable for contracts made by their minor children.; All of the above are true.;

Which of the following will often not constitute valid consideration?

A charitable subscription

Alice offered to sell her computer, monitor and printer to Bradley for $300. Two days later Bradley mails a letter to Alice in which he accepts the offer. Three days later, before Alice has received the letter, Bradley calls to say he won't be able to accept her offer. What is the status of their negotiations?

A contract came into acceptance when Bradley mailed the letter.

Which of the following contracts is VOID?

A contract made by a person who has been adjudicated insane and institutionalized or put under a guardian's care

Which of the following promises are enforceable without consideration?

A contract modification in a contract covered by the U.C.C.; A written offer signed by a merchant to buy or sell goods that assures it will be kept open for one month; A renunciation of a claim a written waiver that is signed and delivered by the aggrieved party when the contract involves a sale of goods; All of the above are enforceable by statute;

Which of the following cannot be disaffirmed until AFTER the age of majority?

A contract that affects title to real estate sold by a minor

Which of the following contracts or contract provisions is most likely to be enforced?

A contract that violates a licensing statute whose aim is to raise revenue

Which of the following would probably be considered to be contrary to public policy?

A contract to bribe a judge; A contract to commit a battery; An agreement not to prosecute a crime; All of the above would be contrary to public policy.;

If the Uniform Commercial Code and the common law of contracts differ, the Uniform Commercial Code would be applied in which of the following situations?

A contract to buy living room furniture

Adams mails an offer to Baker on June 1. Baker receives the offer on June 2. Adams mails a revocation on June 3. Baker mails an acceptance on June 4. Baker receives the revocation on June 5. When was the contract formed?

A contract was formed on June 4.

On April 2, Bonn & Co., CPAs mailed Marble Corp. a signed proposal to perform certain accounting services for Marble provided Marble accepts the proposal by April 30. Under the circumstances

A contract will be formed between the parties if Marble mails an acceptance to Bonn on April 29, even if it is not received by Bonn until May 3.

Which of the following is correct with regard to counteroffers?

A counteroffer operates as a rejection of the original offer.; A conditional acceptance is a common type of counteroffer.; The receipt of a counteroffer terminates the original offer.; All of the above are correct.;

In which of the following situations does the first promise serve as valid consideration for the second promise?

A debtor's promise to pay $500 for a creditor's promise to forgive the balance of a $600 disputed debt

A offers to pay B $10,000 if B will cut down all the trees in a wooded area on A's land. B promises to cut down the trees and then begins work. A reasonable time to complete the job is ten days. After three days B has cut down 50% of the trees. Then, A walks up to B and says: "I revoke." To this, B says "Screw you," and eventually finishes the job eight days after A made his original offer. Which of the following is most true? Assume that an offer for a unilateral contract normally is accepted by full performance of the requested act.

A is bound contractually because, under one of several possible approaches, he could not revoke on these facts, and because B finished the job in time.

Which of the following would likely be considered a non-necessary item for a minor?

A miniature television set to watch the soaps during lunch

Which of the following is enforceable without consideration?

A new promise to pay a debt barred by the statute of limitations

Which of the following is least classifiable as a necessity for which a minor will be held liable on a contract?

A new sports car

Which of the following is an example of unconscionability?

A penalty clause obligating the buyer to pay five times the product's price for failing to accept the goods when delivered

Which of the following is correct with regard to a contract?

A person who deposits money in a bank account has entered into a contract with the bank.

Which of the following would NOT be a merchant under Article 2 of the UCC?

A person who inherits three speedboats and wants to sell them to buy a car

Which of the following would most likely be enforceable?

A promise following the rendering of emergency services that is not supported by new consideration

Which of the following situations are not considered to be sufficient legal consideration?

A promise given in exchange for past occurrence; Where there is a pre-existing duty; Where there is a moral obligation; Where there is an illusory promise; All of the above;

Within the law of consideration, which of the following is a correct statement of law?

A promise in writing to pay a debt barred by the statute of limitations is binding without new consideration.

Which of the following will be legally binding despite lack of consideration?

A promise to donate money to a charity on which the charity relied in incurring large expenditures

Which of the following will be legally binding on all the parties despite the lack of consideration?

A promise to donate money to a charity that was relied upon by the charity in incurring large expenditures.

Which of the following would not be enforceable without consideration?

A promise to pay a debt discharged in bankruptcy where the promise meets the requirements of the Bankruptcy Act

Which of the following would generally be considered to be a revenue raising law?

A statute requiring that salesmen be licensed, but not establishing any educational or training requirements

Which of the following is not a required element of a contract?

A writing

Which of the following is not always necessary in order for a valid contract to be formed?

A writing

Which of the following promises is enforceable in most states?

A written promise to pay a debt barred by the statute of limitations.

The State of Indiana agrees with the Ace Construction Company that Ace will do some repair work on a bridge. The agreement, however, does not specify the exact work to be done, the quality standards Ace must meet, and the time the job must be completed. Also, the agreement states an ambiguous formula for determining Ace's compensation. Later, before either it or the state has done anything, Ace backs out of the job. Then it is sued by the state. Ace defends by arguing that the contract fails because it (or the offer) is indefinite. Which of the following is true?

Ace's argument is correct and the state cannot recover.

Barnes was holding an annual auction at his farm to sell tools, animals, and leftover crops. Adams showed up at the auction and saw a plow among the merchandise. Adams bid $50 for the plow, and no one bid against her. The auctioneer did not accept Adams' bid and stated that the plow would not be sold for such a low price.

Adams' bid was only an offer.

Adele borrowed $1,000 from Beatrice and signed a promissory note due on June 1st. On May 1st, a month before the debt was due Beatrice agreed to accept immediate payment of $800 in full satisfaction of the debt. In January of the following year, Beatrice sought to receive the $200 unpaid balance. What will be the result?

Adele will win because she provided consideration for Beatrice's new promise.

If there is no time specified for the acceptance of an offer, when does the offer terminate?

After a reasonable period of time

X, a manufacturer of pencils, offers to sell Y 1,000,000 pencils for $1 million. The offer, which is signed by X's president, says that it will remain open for six months. The offer does not request that Y pay any consideration for the promise to keep the offer open, and Y does not pay X any consideration. When can X revoke the offer?

After three months

Al Accountant has a tax service and accounting business in Redwood City. He decides to move to Center City, which is 150 miles away and sells his accounting practice to Able and Baker, a CPA firm, and in the sales contract agrees that he will refrain from practicing accounting anywhere within a 100 mile radius of Redwood City for a period of five years. However, on weekends he returns to his house in Redwood City, and when clients call him, he meets with them in his home.

Al is in violation of the sale agreement.

Albert read ElectroCorp's ad in the local newspaper advertising a 4-head VCR for $89. Albert rushed to the store to buy the VCR only to be told by the salesperson that the ad was a misprint and the price should have been $289. Albert gave the salesperson $89 plus sales tax and demanded the VCR.

Albert is merely making an offer to ElectroCorp to buy the VCR for $89 plus sales tax.

Big Bucks, Inc. entered into a contract with Albert Agent under the terms of which Albert would receive $20,000 if he stole trade secrets from the leading competitor of Big Bucks. Albert performed his end of the agreement by delivering the trade secrets. Big Bucks now refuses to pay Albert for his services.

Albert will be unable to recover, because this is an illegal contract.

Generally speaking, three of the following four kinds of statements are treated as invitations to make an offer rather than as offers. One of the four, however, generally is regarded as an offer. Which of the following is most likely to be regarded as an offer?

An advertisement offering a reward for the return of lost property

Which of the following would be considered a valid and legally enforceable agreement?

An agreement by a car salesman not to sell automobiles or automobile parts in Marin County for a period of six months after terminating his employment

Which of the following transactions would be subject to Article 2 of the UCC?

An agreement to sell a television set

Which of the following would be unenforceable because the subject matter is illegal?

An employer's promise not to press embezzlement charges against an employee who agrees to make restitution

Which of the following is true with regard to an exculpatory clause?

An exculpatory clause excuses one party from liability for her own tortuous conduct.; Where one party has a superior bargaining position that has enabled him to impose an exculpatory clause upon the other, the courts are inclined to nullify the provision.; An exculpatory clause may be unenforceable for unconscionability.; All of the above are true.;

In general, which of the following will not terminate an offer?

An inquiry by the offeree as to size or materials

A bank robbery has occurred, and the banker's association has offered a $1,000 reward for information leading to the arrest and conviction of the robber. Several people are claiming to be entitled to the money. Which of them is eligible?

An off-duty deputy sheriff from a county other than the one where the arrest occurred

Which of the following offers terminates earliest? Assume that there is no time limitation on the offer unless the question says otherwise.

An offer to purchase stock on a stock exchange

Which of the following normally is effective upon dispatch?

An offer; A rejection of an offer; A revocation of an offer; Answer = None of the above

Mix offered to sell a parcel of land to Simon for $90,000. The offer was made by Mix in a signed writing and provided that it would not be revoked for five months if Simon promised to pay Mix $250 within 10 days. Simon agreed to do so. Which of the following is correct?

An option contract is formed.

$500 isn't enough money. He refuses to finish the job, unless Betty agrees to pay him $100 more. What law applies to this fact situation?

Andrew was already obligated to paint the house. He gives no additional consideration in return for Betty's promise to pay more money

Anita saw an ad for Marlin's Department Store in which the store advertised word processors for $5. When she went to the store to buy one, the salesclerk told her that the ad was a misprint and should have indicated the price to be $500.

Anita is making an offer to buy the word processor.

On April 1, A makes a valid offer to B. A mails B a revocation on April 2, which B receives on April 4. In the meantime, B mails A a rejection on April 3, which reaches A on April 5. When did this offer terminate?

April 4

Arthur contracts to build a garage for Bob for a price of $6,000. Because of an increase in the cost of labor and materials, Arthur refuses to perform. Bob wants the garage, so he agrees to pay an additional $500.

Arthur has given no additional consideration, and under the common law must perform at the agreed upon original price.

Which of the following would most likely be a merchant with respect to the goods in question under the Code definition?

Arthur is an authorized IBM computer dealer.; Brian employs two salesmen to sell his homemade furniture.; Clarence has a store in which he sells used lawnmowers.; All of the above would be merchants.;

Edna, who runs a flower shop, sells Jim a shotgun. Jim is unemployed, and Edna has never before sold a firearm in her life. Which of the following is true?

Article 2 of the UCC applies because a shotgun is a personal property.

Able made a public offer of a reward for information leading to the arrest of a thief. Without knowledge of the reward, Baker furnished information to the police that resulted in arrest of the thief. Which is correct?

Baker could not accept the offer without knowledge of its existence.

Why does it matter whether or not a contracting party is an Article 2 merchant?

Because Article 2 sometimes creates different rules for merchants in sale-of-goods cases.

Dad wants to give his daughter, Priscilla, a new BMW 540i (worth perhaps $50,000) upon her graduation from high school. Priscilla says: "Let's make it a contract, Dad." So Dad promises to give Priscilla 540i when she graduates, in exchange for Priscilla's promise to pay Dad $1. There is no contract here. Why?

Because a genuine bargained-for exchange is not present here

Why is past consideration not consideration?

Because it is not bargained-for exchange

Why is nominal consideration usually not consideration?

Because it suggests the absence of a genuine bargain

Bill Businessman agrees to sell two different goods to his friend, Ron Retailer. One item is legal, and one item is illegal. The contract price is $2,000.

Bill may recover for the legal item, but he may not recover for the illegal item.

Bill and Mary are merchants. Bill orders 100 computers from Mary which she accepts. In the order he specifies that he wants model ABC. Mary, a long-time friend to Bill, knows what the computers will be used for and substitutes Model XYZ, computer with similar capabilities, but not exactly the same. Bill objects within a reasonable period of time to the substitution as a material change.

Bill's original order must be filled.

Arthur has offered to take Bob into his accounting firm as a partner upon payment of $5,000 cash. In response, Bob says, "I'll give you $3,000 cash now and I will pay you the remainder in two months after I see whether things are working out as a partnership."

Bob has made a counteroffer and hence there is no contract.

Anna by mistake delivers to Bob a plain, unaddressed envelope containing $50 intended for Cora.

Bob's obligation to return the money is quasi contractual.

Jack told Abe that Jack would pay Abe $10, if Abe washed Jack's car. Abe washed the car. At that point:

Both Jack and Abe have given consideration.

Wilcox mailed Norriss an unsigned contract for the purchase of a tract of real property. The contract represented the oral understanding of the parties as to the purchase price, closing date, type of deed, and other details. It called for payment in full in advance or certified check at the closing. Norriss signed the contract, but added above his signature the following: This contract is subject to my (Norriss) being able to obtain conventional mortgage financing of $100,000 at 13% or less interest for a period of not less than 25 years. Which of the following is correct?

By adding the "conventional mortgage" language above his signature, Norriss created a condition precedent to his contractual obligation and made a counteroffer.

Which of the following is least classifiable as a necessity for which a minor will be held liable on a contract?

Cassette tapes

Nix sent Castor a letter offering to employ Castor as controller of Nix's automobile dealership. Castor received the letter on February 19. The letter provided that Castor would have until February 23 to consider the offer and, in the meantime, Nix would not withdraw it. On February 20, Nix, after reconsidering the offer to Castor, decided to offer the job to Vick, who accepted immediately. That same day, Nix called Castor and revoked the offer. Castor told Nix that an acceptance of Nix's offer was mailed on February 19. Under the circumstances,

Castor's acceptance was effective when mailed.

Charlie thought that he would need $5 to buy a concert ticket, so he entered a contract with Laura to mow her lawn for $5. When Charlie was almost finished mowing the lawn, his friend Tony came by and told Charlie that the cost of the concert ticket would be $10. Charlie stopped mowing the lawn and told Laura that he would not finish unless Laura agreed to pay him $10. If Laura agrees, and Charlie finishes the job:

Charlie may not collect more than $5, because he had a contractual duty to mow the lawn for $5.

Cindy Smith, age 16, buys a Chevy Camaro from Mike Mason, age 23, for $4,000. Cindy's parents, who give her everything she wants, loaned her the money for the car. The reason for Cindy's purchase is that all her friends have cars and she feels left out without one. One week after buying the car, however, Cindy changes her mind and told Mike that she wants to disaffirm the contract. When Mike comes to pick up the car and give Cindy her money, though, Cindy changes her mind again, telling Mike that "I'll stick by the deal." But when Cindy's parents gave her a new car for her seventeenth birthday, she finally decides to disaffirm once again. Which of the following is true?

Cindy can still disaffirm.

Which of the following is a type of counteroffer?

Conditional acceptance

The Acme Corporation is having a "T.V. giveaway." To win, a person must guess the number of marbles in a large jar. No purchase is necessary. This is probably not an illegal lottery because what element is lacking?

Consideration

Which of the following is a true statement about the requirements of an assignment?

Consideration is not required.

A ______ is an obligation imposed by law to avoid injustice.

Contract Implied in Law; Quasi Contract;

In response to an offer, an acceptance is mailed on day 1, followed by a rejection on day 2, rejection arrives on day 3 and acceptance arrives on day 4. Which of the following is true?

Contract is formed on day 1.

In response to an offer, a rejection is mailed on day 1, followed by an acceptance on day 2, the rejection arrived on day 4, the acceptance arrived on day 3. Which of the following is true?

Contract is formed on day 3.

If a covenant not to compete is found to be against public policy, the courts may do which one of the following?

Delete the unreasonable parts

Harold offers to sell his diamond ring to Emily. Diane overhears the offer and says, "I accept the offer."

Diane cannot accept the offer, because it wasn't made to her.

On which of the following would the usury statutes usually apply?

Direct loan from a bank to borrower

Don, a minor, contracts to sell 100 acres of land to Jerry who is also a minor. Don reaches the age of 18 as does Jerry. Don takes no action to avoid the contract for several months. Which of the following most accurately describes the status?

Don has ratified the contract.

Donald, a minor, makes a contract to sell a motorcycle to Albert, an adult. One week later, Donald has his eighteenth birthday and shortly thereafter Albert picks up the motorcycle.

Donald has expressly ratified the contract.

In which of the following instances will silence by the offeree constitute acceptance?

During the course of prior business dealing, the offeree has always sent a rejection if the items were not wanted. The offeror always shipped the items if such a rejection was not received.

Dye sent Hill a written offer to sell a tract of land for $60,000. They were engaged in a separate dispute. The offer stated that it would be irrevocable for 60 days if Hill would promise to refrain from suing Dye during this time. Hill promptly delivered a promise not to sue during the term of the offer. Dye subsequently decided that the possible suit by Hill was groundless. Dye then phoned Hill and revoked the offer. Hill mailed an acceptance. Dye did not reply. Under the circumstances,

Dye's offer was supported by consideration and was not revocable when accepted.

Elmer wrote a letter to his friend Fred offering to sell Fred an 80 acre farm for $200,000. After mailing the letter, Elmer learns that the farm is actually worth $300,000 and changes his mind about selling.

Elmer can revoke his offer at any time before acceptance, because there is no consideration to keep it open.

______ is a provision excusing one party from fault or liability

Exculpatory clause

Samuel Tate enters into a contract with Bill Smith under the terms of which Smith is to pay Tate $7,000 and Tate is to build a garage, repair a boat, and build a doghouse. If the doghouse has not yet been built, which term describes the type of contract in existence?

Executory contract

Which of the following is necessary for the existence of an offer?

Facts and circumstances that reasonably indicate the offeror's intent to contract

Which of the following is LEAST likely to constitute ratification of a contract made by a minor?

Failing to disaffirm a completely executory contract within one month after the age of majority

A valid contract may be unenforceable for which reason(s)?

Failure to satisfy the statute of frauds; Running of the statute of limitations;

Which of the following is not generally required in order to have a valid contract?

Fairness of the bargain

A binding promise or agreement requires that the parties to the agreement have contractual capacity.

False

A debt which has been reduced to a sum certain of money is called an unliquidated debt.

False

A minor is liable for necessities and for the agreed price.

False

A minor is not liable for his torts.

False

A minor may disaffirm a conveyance of land prior to reaching majority or within a reasonable time after doing so.

False

If an offer is made to the public, a person can accept that offer even though he has no knowledge that it has been made.

False

Legal sufficiency of consideration and adequacy of consideration are virtually synonymous terms.

False

One of the most frequently occurring applications of the doctrine of promissory estoppel is to charitable subscriptions.

False

The Code follows the common law rule and allows the parties to a contract to modify that contract without additional consideration.

False

The Uniform Commercial Code has not changed the traditional common law view that a minor could avoid her contract and recover property, even though a third party may have given value for it and had no notice that it was the property of a minor.

False

Today, under the UCC, a person buying goods from a minor may not transfer valid title to a good faith purchaser for value.

False

Fay, age 17, ordered a pair of skis on the installment plan. She paid $20 every month until she turned 18, the age of majority. The next day, she sold them to Sharon and disaffirmed the contract. What result?

Fay is still liable because selling the skis amounts to a ratification.

Cara offered to pay Peter to paint a house located at 103 Canton Street, and he accepted. However, Peter incorrectly wrote down the address as 108 Canton Street. As a result, he painted the wrong house. Fred Fibs, the owner of 108 Canton Street, say Peter painting his house but said nothing. When the job was completed, Peter tried to collect the contract price. He will be able to recover from

Fibs under the theory of quasi-contract.

Which of the following will support a contract?

Forbearance to do an act

On February 12, Harris sent Fresno a written offer to purchase Fresno's land. The offer included the following provision: "Acceptance of this offer must be by registered or certified mail, received by Harris no later than February 18 by 5:00 p.m. CST." On February 18, Fresno sent Harris a letter accepting the offer by private overnight delivery service. Harris received the letter on February 19. Which of the following statements is correct?

Fresno's letter constituted a counteroffer.

Gudrun owned a 2,000-acre country estate. She signed a written agreement with Johann, selling the house on the property and "a sufficient amount of land surrounding the house to create a park." The price was stated to be $200,000. When Gudrun refused to honor the agreement, Johann sued.

Gudrun will win because the agreement is not reasonably definite.

Harold ordered an aluminum storm door from Sears for $249.99. Before it was delivered, Sears ran an ad in the paper for the same storm door at $179.99. Harold calls Sears and demands the advertised price. They say okay.

Harold must pay $179.99.

Harper is opening a small retailing business in Hometown, U.S.A. To announce her grand opening, Harper places an advertisement in the newspaper quoting sales prices on certain items in stock. Many local residents come in and make purchases. Harper's grand opening is such a huge success that she is unable to totally satisfy the demand of the customers. Which of the following correctly applies to the situation?

Harper has made an invitation seeking offers.

Joe Lowe is a realtor in Mississippi. He is duly licensed in that state. He contracts to sell and does successfully sell 1000 acres of land belonging to Greer. This land is located in both Mississippi and Tennessee. The land constitutes one tract; 800 acres of the tract are located in Mississippi and 200 acres of the tract are located in Tennessee. Upon selling the property, Mr. Lowe was to receive a $14,000 commission. Which of the following is most correct?

He is entitled to the commission based on the land sold in Mississippi only.

Sally Smith loses her pet Persian cat. She posts a $100 reward offer. This offer is made to the public generally. Bill Jones finds the cat. Which of the following is most correct concerning his ability to claim the reward?

He may not collect the reward unless he knew of the reward.

Norm Hefty was a door-to-door salesman for Toothrot Candy Company. His duties were to call on customers and make sales in Bedford, Indiana, which is located in the southern part of the state. Toothrot's business territory covers the entire state of Indiana. Hefty and Toothrot had entered into a written employment contract at the time he joined the firm. One of the clauses in the contract stated that if Hefty ceased working for Toothrot, he could not work as a salesman for a competing company anywhere in Indiana for a period of five years. Hefty left his employment with Toothrot and promptly went to work as a salesman for a candy company that competed with Toothrot. His duties with the new employer were confined to the town of Elkhart, which is located in northern Indiana. Toothrot has sued Hefty in an effort to obtain an injunction against further violation of the non-competition clause in the parties' employment contract. Which of the following is most true?

Hefty will win, because the area of the restriction is not reasonable.

Henry gives Big Al $50 in return for Al's promise to assault Sara. Henry hopes Sara will come running back to him if she is frightened by Al. Sara never comes back after the assault and Henry finds out Al went off drinking with the $50. Which is true?

Henry can neither get the $50 back nor force Al to act.

Alice hands Henry a written offer to buy Henry's camera for $400. In which of the following situations would the court find that there is a rejection?

Henry tears up the written offer in front of Alice and says, "That's what I think of your offer."; Henry says nothing, but in front of Alice turns to his son, Chris, and says, "Here, this camera is yours."; Henry says to Alice, "I accept, provided you pay me $475 instead.";

To announce the grand opening of a new retail business, Hudson placed an advertisement in a local newspaper quoting sales prices on certain items in stock. The grand opening was so successful that Hudson was unable to totally satisfy customer demands. Which of the following statements is correct?

Hudson made an invitation seeking offers.

Harold purchased 400 pairs of gloves from Isaac at a contract price of $800. Fifty of the gloves were defective and a dispute arose as to the amount due and owing under the contract. Harold refuses to pay the $800 and Isaac is threatening to sue. Which of the following is correct with regard to this transaction?

If Isaac agrees to accept $600 to settle the dispute and Harold agrees to pay that amount, the agreement is enforceable.

Ann Mayer wrote Tom Jackson and offered to sell Jackson a building for $200,000. The offer stated it would expire 30 days from July 1. Mayer changed her mind and does not wish to be bound by the offer. If a legal dispute arises between the parties regarding whether there has been a valid acceptance of the offer, which of the following is correct?

If Jackson categorically rejects the offer on July 10th, Jackson cannot validly accept within the remaining stated period of time.

Which of the following is true regarding gambling statutes?

If a purchase is required, a grocery store drawing will be an illegal lottery.

Seventeen-year-old Tim Teenager decides he would like a new car, so he brings his 1972 Ford Maverick to the local Ford dealer as a trade-in on a new Thunderbird. He turns over his Maverick to the dealer and drives away with a new Thunderbird. Two days later, Tim decides he cannot afford the payments for the new car on his salary as a burgermaker, so he returns the car and asks for his Maverick back. The dealer says he will take the Thunderbird and rescind the contract, but that he is unable to return the Maverick because he sold it to another buyer.

If the dealer sold the car to a good faith purchaser, then Tim has no right to get his Maverick back.

If agreed to by both debtor and creditor, in which of the following situations will a smaller sum be unable to discharge a larger debt?

If the smaller sum is paid when the larger sum would ordinarily be due, but the debtor is on the verge of insolvency

Which of the following is correct with regard to consideration?

In a unilateral contract, a promise is exchanged for an act or forbearance to act.; In a bilateral contract, there is an exchange of promises.;

When Lynx, Inc. hired Parr, Parr signed an employment contract prohibiting Parr from competing with Lynx during and after employment. While employed, Parr acquired knowledge of many of Lynx's trade secrets. Which of the following statements is correct?

In determining whether Parr may compete with Lynx, the court should consider, among other factors, whether the agreement is necessary to protect Lynx's legitimate business interests.

A and B agree that A will sell B all B's requirements of coal at a stated price per ton. This agreement:

Is binding, so long as B's requirements are in good faith, and are not disproportionate to any estimate in the contract or to B's normal previous requirements.

A and B have a written contract whereby A agrees to sell B a plot of land for $100,000. Later, without terminating the first contract, the parties modify the deal so that A sells B the same plot of land for $125,000. The second agreement:

Is not a contract because there is no consideration for B's promise

Julian Fagan is owed $10,000 by Larry Berry. Julian has insisted that Larry pay this amount for a period in excess of a year. Larry has refused to pay. Julian writes Larry a letter in which he states, "If you do not pay the $10,000 by December 22, 1983, I shall file suit against you and collect interest and court costs." What is the status of such a threat?

It is perfectly legal.

Jason's mother would like him to go to college, so in June he enrolls at Ivory Towers State University. He also quits his job and tells his mother his plans to continue taking classes. His mother says, "I'm so happy that you are going to college that I want to pay for your books." Jason then sends her a bill for $485. Which of the following is true regarding his mother's promise?

It is unenforceable, because Jason has already enrolled in school and there is no consideration.

Three days before she was judicially declared mentally incompetent and institutionalized, Irma bought a $50,000 automobile for cash and took delivery of it. Which of the following is true about this contract? Assume that Irma was pretty far gone when she bought the car.

It is voidable.

UCC section 2-207 (the "battle of the forms" provision) implicitly distinguishes between different terms in the acceptance and additional terms in the acceptance. The latter refers to a subject not mentioned in the offer, while in the former case the acceptance treats some subject differently than the offer does. Under 2-207, why does this distinction matter?

It matters because, while additional terms can become part of the contract, it isn't clear what terms apply in "different terms" situations.

Which of the following is true regarding promissory estoppel?

It requires that the promisor should reasonably expect to induct the promisee's action or forbearance.

Sparky, Nancy's pet dog was lost. Nancy ran an ad in a local paper offering $50 for Sparky's return. Jane did not know of the advertisement but found Sparky, and because Nancy's address was on Sparky's collar returned the dog to Nancy.

Jane could not accept Nancy's offer because she did not know of it.

While Jane is swimming she cuts her foot on some coral and is quickly surrounded by several sharks. Within earshot of several startled sunbathers, Jane screams that she will pay one million dollars to anyone who saves her. Gertrude, the lifeguard on duty, throws Jane a line and pulls her in. When Jane refuses to pay, Gertrude sues. The court would find for:

Jane, since Gertrude was under a pre-existing duty to do as she did.

On April 2, Jet Co. wrote Ard, offering to buy Ard's building for $350,000. The offer contained all of the essential terms to form a binding contract and was duly signed by Jet's president. It further provided that the offer would remain open until May 30 and an acceptance would not be effective until received by Jet. On April 10, Ard accepted Jet's offer by mail. The acceptance was received by Jet on April 14. 1. Assume that on April 11 Jet sent a telegram to Ard revoking its offer and that Ard received the telegram on April 12. Under the circumstances,

Jet's revocation effectively terminated its offer on April 12.

On December 3, Joncie mailed a letter to Stumos offering to buy her stereo for $1,000 upon her delivering it to Joncie's home on or before December 24. Stumos immediately sent a letter to Joncie accepting the offer and promising to make delivery. What is the significance of the letter sent by Stumos?

Joncie could revoke the offer after receiving the letter.

Ken promises not to foreclose on a mortgage that he holds on an office complex that Christopher owns. In reliance on this promise, Christopher expends $200,000 to remodel the complex. Which of the following is correct with regard to Ken's promise?

Ken's promise is not to foreclose is unsupported by consideration.; Ken's promise is noncontractual.; Ken's promise will be enforced against him based upon the doctrine of promissory estoppel.;

Kent, a 16-year-old, purchased a used car from Mint Motors, Inc. Ten months later, the car was stolen and never recovered. Which of the following statements is correct?

Kent may disaffirm the purchase because Kent is a minor.

Lydia promised to pay Lavinia $10,000 if she refrained from smoking for 1 year. Lavinia refrained and brought suit when Lydia failed to pay. Who will win?

Lavinia will win because Lydia received a legal benefit.

Which of the following statements is correct?

Legal sufficiency has nothing to do with adequacy.

Sam Student was hit by a car crossing the street and knocked unconscious. Fast Ambulance Service took him to the hospital while he was sill unconscious. What is Sam's liability for Fast Ambulance's fee?

Liable under quasi-contract (implied-in-law contract) theory

Generally, minors are liable for

Libel; Negligence; Fraud;

A scheme for the distribution of property by chance among persons who have paid or agreed to pay a valuable consideration for the chance is a

Lottery

Marianne promises to pay Nils $2,500 for one of his original sculptures, if she receives the $3,000 she anticipates receiving from her mother for her birthday.

Marianne has made a conditional promise.

Mark paid off his brother Steve's debt to the loan shark on condition that Steve wouldn't get a pair of cement shoes for Christmas. The loan shark says that he doesn't have to honor that promise because Steve didn't pay. Mark would help his cause by accurately pointing out which of the following to the loan shark?

Mark's payment to the shark was a legal detriment to Mark.

On January 1, Lemon wrote Gina Martin offering to sell Martin a ranch for $80,000 cash. Lemon's letter indicated that the offer would remain open until February 15 if Martin mailed $100 by January 10. On January 5, Martin mailed $100 to Lemon. On January 30, Martin telephoned Lemon stating that she would be willing to pay $60,000 for the ranch. Lemon refused to sell at that price and immediately placed the ranch on the open market. On February 6, Martin mailed Lemon a letter accepting the original offer to buy the ranch at $80,000. The following day, Lemon received Martin's acceptance. At that time the ranch was on the market for $100,000. Which of the following is correct?

Martin's call on January 30 automatically terminated the January 1 offer.

Martin Stores, Inc., decided to sell a portion of its eight-acre property. Consequently, the president of Martin wrote several prospective buyers the following letter: Dear Sir: We are sending this notice to several prospective buyers because we are interested in selling four acres of our property located in downtown Metropolis. If you are interested, please communicate with me at the above address. Don't bother to reply unless you are thinking in terms of at least $100,000. James Martin, President Under the circumstances, which of the following is correct?

Martin's communication did not constitute an offer to sell.

Mary, a self-employed 16-year-old whose parents are dead, buys a dress on credit for $50. After receiving the dress and discovering that its reasonable value is only $25, Mary tries to disaffirm the deal before paying the $50 (and while she is still 16). In this case:

Mary cannot disaffirm, but she is only bound to pay $25

Bill and Mary are merchants. Bill orders 100 computers from Mary which she accepts. In the order he specifies that he wants model ABC. Mary, a long-time friend to Bill, knows what the computers will be used for and substitutes Model XYZ, computer with similar capabilities, but not exactly the same. If Bill had stated in the original offer "no substitutions will be accepted"

Mary is bound to the contract and must provide Bill with Model ABC computers.

Bill and Mary are merchants. Bill orders 100 computers from Mary which she accepts. In the order he specifies that he wants model ABC. Mary, a long-time friend to Bill, knows what the computers will be used for and substitutes Model XYZ, computer with similar capabilities, but not exactly the same. If Bill and Mary were NOT merchants

Mary's acceptance was not an acceptance, but a counter-offer.

Which of the following requires consideration in order to be binding upon the parties?

Material alteration of a personal service contract

Which of the following requires consideration to be binding on the parties?

Material modification of a contract involving the sale of real estate

May 1, A hands a written offer to B for the sale of A's car. On May 2, A mails B a letter revoking the offer. On May 3, A calls and tells B that he is revoking. On May 4, B learns that A has sold the car to C. On May 5, A's letter finally gets to B. A's offer terminated on:

May 3

Mike Minor buys some real estate for investment purposes. The contract obligates Mike to make monthly installment payments for ten years. Mike reaches the age of majority one month after making the contract. After this, Mike makes 25 monthly payments under the contract, but then decides that he wants to disaffirm the deal. Which of the following is MOST true?

Mike cannot disaffirm because he has already ratified the contract.

Mildred saw a vase in an antique shop. A tag on the vase said "Genuine Chinese Vase, $125." Mildred said to the owner of the shop, "I'll buy this vase for $125." Milford, the owner of the shop, refused to sell the vase. In a lawsuit brought by Mildred against Milford,

Milford will win because he rejected Mildred's offer.

The elements of an enforceable contract include all but which of the following?

Mutual promises

If Nora had paid Nick $1 to hold the offer open for a week:

Nick may not revoke since Nora has taken an option on the bracelet.

On April 1, Fine Corp. faxed Moss an offer to purchase Moss' warehouse for $500,000. The offer stated that it would remain open only until April 4 and that acceptance must be received to be effective. Moss sent an acceptance on April 4 by overnight mail and Fine received it on April 5. Which of the following statements is correct?

No contract was formed because Fine received Moss' acceptance after April 4.

On May 1, Wendy posted the following notice on a campus bulletin board: NEEDED: Tutor for Business Law I. I don't know a tort from a torte. $10.00 an hour. Call Wendy, 555-3335, evenings. On May 4, Sarah saw the sign and left a message with Wendy's roommate that Sarah accepted Wendy's offer. On may 5, Wendy wrote in red ink across the sign, "Forget it! Dropped the class." Later that evening Wendy received Sarah's message.

No contract was formed because no offer was made by Wendy. Advertisement is not an offer.

An offeree is generally under what legal duty to reply to an offer?

No legal duty

The Save the Earth Society mailed unsolicited a calendar to all graduates of U.S. West University, including Sidney, with a statement that, if the recipient did not return the calendar within 10 days, he or she would be billed $15. Sidney will be liable for the $15 if he uses it or throws it out?

No, Silence is not Acceptance.

On September 27, Summers sent Fox a letter offering to sell Fox a vacation home for $150,000. On October 2, Fox replied by mail agreeing to buy the home for $145,000. Summers did not reply to Fox. Do Fox and Summers have a binding contract?

No, because Fox's letter was a counteroffer.

On November 1, Yost sent a telegram to Zen offering to sell a rare vase. The offer required that Zen's acceptance telegram be sent on or before 5:00 p.m. on November 2. On November 2, at 3:00 p.m., Zen sent an acceptance by overnight mail. It did not reach Yost until November 5. Yost refused to complete the sale to Zen. Is there an enforceable contract?

No, because Zen did not accept by telegram.

Harry's mother, Harriet, calls her son upon the birth of his daughter, Hariette. Mom says, "I'm so happy that you have named the baby after me that I'm going to send you $100 for her!" No money came. Is the promise enforceable?

No, because there is no bargained-for exchange.

A week before the concert, Joan told you she probably would not be able to use her concert tickets. Later that day, Joan wrote you a note offering to sell you the tickets for $50 and gave it to Betsy to deliver. Betsy stuck it in her purse and forgot about it. You called Joan the day of the concert, but she already sold the tickets. Is Joan liable to you for their value?

No, because there was no offer since it was never communicated.

Mary, age 17, sold Mark, age 22, the briefcase she got for graduation. Mark's father liked it and bought it from him. If Mary decides to disaffirm the contract, will Mark's father have to return the briefcase to her?

No, if Mark's father bought it without knowing from whom his purchased it.

Mix entered into a contract with Small that provided for Small to receive $10,000 if he stole trade secrets from Mix's competition. Small performed his part of the contract by delivering the trade secrets to Mix. Mix refuses to pay Small for his services. Under what theory may Small recover?

None, because of the illegal nature of the contract.

Nick, a used car dealer, verbally offers to sell a diamond bracelet to Nora for $100. Nora asks Nick if she can have a few days to think it over. Nick tells her that she can have up to a week to consider the offer and promises not to revoke or to sell to anyone else during that period. The next morning when Nick sees Nora coming up his front walk, he quickly opens a window and shouts to Nora that he is revoking the offer. Nora then tenders $100 and demands the bracelet. When Nick refuses, Nora brings suit:

Nora will lose because, despite his promise, Nick could revoke his offer.

What standard is used when determining the intent of an offer?

Objective

Joe Minorca purchased a motorcycle from Big Rig Company on May 1. Joe's birthday is June 17, at which time he will have attained his majority. Which of the following actions is least likely to be considered evidence as a ratification of the contract of purchase?

On June 16, Joe made an installment payment.

Gus Parker owned a race horse that had not made a good showing in the last 10 races. Disgusted with the horse, Parker stated that he would sell the horse for $1.00. Sam Hood was in the group to whom Parker was speaking and immediately said that he would accept the horse for $1.00. Parker agreed to accept $1.00 after the afternoon's race in which Parker was obligated to participate. In the afternoon's race, the horse won and Parker decided that he did not really want to sell the horse for $1.00.

One dollar constitutes sufficient consideration.

Which of the following represents the basic distinction between a bilateral contract and a unilateral contract?

Only one promise is involved if the contract is unilateral but two if it is bilateral.

To be effective, an offer must be definite and complete. Which of the following is correct?

Only reasonable definiteness is necessary.

Which of the following statements is most descriptive of past consideration?

Past consideration is no consideration.

Dan Doan negligently injured Paul. Paul told Doan that he would agree not to sue Doan if Doan promised to pay $1000.

Paul is bound by his promise since he received consideration for his promise not to sue.

Tom Payne had a toothache and stopped by Quick Fix Dentist during his lunch hour. Although he had never been to Quick Fix before, Fix accepts Payne as a patient and pulls a tooth. On Payne's way out, the receptionist hands him a bill for $500. Payne is outraged and claims that a contract was never formed.

Payne is only liable for a reasonable amount.

Percy is a rich boy whose parents supply him with every conceivable necessity of life. While still a minor, Percy buys a coat on credit from a men's store for $5000. After wearing the coat for a while, Percy decides that it bores him and that he'd like to disaffirm. Which of the following is MOST likely to be true? Assume that Percy is still a minor.

Percy can disaffirm the contract, and he must return the coat.

Article 2 of the Uniform Commercial Code deals with what type of property?

Personal

______ is/are any property other than an interest in real property

Personal Property

Who makes the offer in an auction?

Persons attending auction

On June 15, Peters orally offered to sell a used lawn mower to Mason for $125. Peters specified that Mason had until June 20 to accept the offer. On June 16, Peters received an offer to purchase the lawn mower for $150 from Bronson, Mason's neighbor, Peters accepted Bronson's offer. On June 17, Mason saw Bronson using the lawn mower and was told the mower had been sold to Bronson. Mason immediately wrote to Peters to accept the June 15 offer. Which of the following statements is correct?

Peters' offer had been revoked and Mason's acceptance was ineffective.

Which of the following is an exception to the consideration requirement?

Promissory estoppel; Promises to pay debts whose enforcement is barred by the statute of limitations; Charitable subscriptions; All of the above;

John Fourcade orders a new Lincoln Continental from Belk Ford of Oxford, Mississippi. Belk Ford receives the new car and services it. They erroneously deliver it to Breck Tyler. When Breck discovers that he has received the car, which one of the following best describes the contract situation that exists between Breck Tyler and Belk Ford?

Quasi

Randy, a minor, buys a new four-wheel-drive truck from Jones Ford. Randy sells this truck to his cousin, Steve, who is an adult. Steve conveys this vehicle to Arthur Smith. Arthur does not personally known Steve or Randy. Which of the following expresses the status of this situation?

Randy may not recover the vehicle from Mr. Smith.

Stable Corp. offered in a signed writing to sell Mix an office building for $350,000. The offer, which was sent by Stable on April 1, indicated that it would remain open until July 9. On July 5, Mix mailed a letter rejecting Stable's offer. On July 6, Mix sent a telegram to Stable accepting the original offer. The letter of rejection was received by Stable on July 8 and the telegram of acceptance was received by Stable on July 7. Which of the following is correct?

Receipt of Mix's telegram resulted in the formation of a valid contract.

If restriction in an employment contract is found to be too harsh, a court may do any of the following EXCEPT which?

Refuse to void the restriction

Robert Briscoe is 17 years old. He lies to Belk Ford about his age, saying he is 18 in order to induce them to sell him a new pickup. Belk falls for this lie and sells him the pickup. Which of the following is correct?

Robert may not disaffirm since he lied.

Brian makes a material misrepresentation of fact regarding his horse to Rosalind while out riding one day. Later that day, Rosalind makes an offer to buy the horse. Brian accepts without correcting his earlier misrepresentation of fact.

Rosalind may avoid the contract.

S agrees to sell a used typewriter to B for $1. The market value is $250.00. B accepts. Both the offer and acceptance are in writing.

S is bound because the law doesn't inquire into the fairness or adequacy of consideration.

Which of the following would be an invalid ratification?

Seventeen year old Tim Teenager ratifies a contract which he signed two months ago.

After making her bid at an auction, Alice wishes to withdraw her bid before the auctioneer's hammer falls:

She may do so since, generally, offers can be revoked any time before acceptance.

The Rogers family has always wanted to buy the beautiful house at the top of the hill. The owners of the house, the Thompsons, decided to sell and called Mr. Rogers. Before he could get back in touch with Mr. Thompson, he suffered a heart attack and died. Mrs. Rogers still wants the house.

She may not accept since the offer is terminated.

Sheila contracts with Margie whereby Margie agrees to construct a house for Sheila for $600,000. When the house is partially built Margie says she is unable to complete the house for the agreed on price because of an unexpected increase in the cost of building materials. Sheila agrees to pay an additional $50,000. When the house is completed, Sheila refuses to pay more than $600,000 and Margie sues. If, in the example above, in return for Sheila's promises to pay the $50,000 extra, Margie promises to put in a double, instead of a single, driveway ($1,500 difference in value), when Margie sues for the additional $50,000, most likely the court would rule that:

Sheila has to pay the $50,000 since the pre-existing duty rule would not cover situations in which she is receiving something she was not entitled to under the original agreement.

Sheila contracts with Margie whereby Margie agrees to construct a house for Sheila for $600,000. When the house is partially built Margie says she is unable to complete the house for the agreed on price because of an unexpected increase in the cost of building materials. Sheila agrees to pay an additional $50,000. When the house is completed, Sheila refuses to pay more than $600,000 and Margie sues. Most likely the court would hold that:

Sheila wins because of the pre-existing duty rule.

Which of the following is untrue regarding the contracts of intoxicated persons?

Slight intoxication will destroy one's contractual capacity.

In violation of a state licensing statute, Jones purports to be an attorney. After making that allegation, he contracts to perform legal service for Smith. Smith then pays Jones a $500 retainer. Later, after discovering that Jones is not licensed and therefore cannot get the job done, Smith sues Jones for the $500. What is the most likely result, and why?

Smith probably wins, because parties for whose protection a regulatory statute has been enacted often can recover amounts paid under a contract declared illegal by the statute.

Which of the following is true regarding exculpatory clauses in contracts?

So long as there is equal bargaining power and voluntary consent, such contracts will be enforced.; So long as the terms of the contract are conspicuously disclosed and clearly known, such contracts will be enforced.; Such clauses can be used to disclaim international tort liability.; Answer = None of the above.

Steve purchases a four-wheel-drive truck from Belk Ford. Steve is only 17 years of age. He wrecks the vehicle and attempts to disaffirm the contract and have Belk Ford repay him all that he has paid. What would happen?

Steve may simply return the vehicle and get his money.

Steven offered to sell his portable CD player to Thomas for $50, and told Thomas that he had ten days in which to accept. One week later Thomas saw the CD player at Violet's house. Violet tells Thomas that she just bought it from Steven.

Steven has revoked his offer to Thomas.

Which of the following is most true regarding non-competition clauses in contracts?

Such clauses are less likely to be enforced when they occur in an employment contract.

Which of the following is true regarding the approach taken by the UCC toward the proper means of acceptance and the implications of using it or not using it?

The UCC does not change the traditional rule that, where the offer stipulates a certain means of acceptance, the acceptance must use that means or there is no contract.

A makes an offer to sell B a computer by testing B the offer. The offer says nothing about authorized or stipulated means of acceptance. Which of the following is true?

The acceptance is effective upon dispatch if it is by any reasonable means of communication.

Michelle's Boutique places an ad in the Sunday paper for beautiful, top-of-the-line designer suits for $3.00. Irene sees the ad in the paper and goes to the store to stock up on business suits for her new job. Michelle apologizes for the misprint. Irene has just finished a class in contract law and insists that the store sell her 5 suits for $15.00. Irene threatens to sue Michelle for breach of contract.

The ad in the newspaper is a solicitation seeking offers, but is not an offer to sell; therefore, Irene will not be able to successfully sue for breach of contract.

Marilyn heard an announcement made at her school offering a thousand dollar swimming scholarship to anyone who could swim 500 laps in the school pool. Marilyn began swimming. She has reached lap number 460; she feels great and is sure she can make it all the way.

The advertiser must permit Marilyn the opportunity to finish her attempt to swim the 500 laps.

A and B contract for the sale of goods. Later A, who is 13 years old, wants to cancel the contract on grounds of incapacity. Which of the following is true? Assume that neither A nor B is a merchant.

The common law capacity rules apply, because Article 2 of the UCC has no rules on the subject

Harry promised to sell his guitar to Harriet, who promised to pay him $1,000. After Harriet tendered payment, Harry reneged on his promise, so Harriet filed suit. Which of the following is correct?

The consideration for Harry's promise is sufficient.

Darla offers to pay Edward $6,000 for Edward's car, provided that Darla receives that much from her uncle's estate, which is currently being probated. She expects to know for sure how much she will receive within a week or so.

The consideration moving from Edward to Darla is the transfer of title to the car.; The consideration moving from Darla to Edward is the promise of $6,000 subject to a condition.;

M, a minor, contracted with A, an adult. The contract has been fully performed by both. M reaches majority plus 3 months.

The contract is ratified because of a failure to disaffirm within a reasonable time.

Elvis makes an offer to Fred, but before Fred can accept, the state supreme court decides a case that makes Elvis's offer illegal. What is the effect of the court decision on the offer?

The court decision automatically terminates the offer.

Phil Fairbanks was approached by Nickle Corporation to write the history of Nickle for $15,000. The president of Nickle told Fairbanks the job was his if he would agree to cleverly defame its leading competitor, Mogul Corporation, using sly innuendo and clever distortion of the facts. Fairbanks wrote the history. It turned out that the Mogul passages were neither sly nor clever, although they were defamatory, and Mogul obtained a judgment against Nickle. Fairbanks is seeking to collect the final $5,000 installment of the contract. Nickle refuses to pay and seeks to recover the $10,000 it has paid. In the event of a lawsuit

The court will deny relief to either Fairbanks or Nickle.

In Florida which of the following is true about situations where a minor misrepresents his age to get an adult to contract with him?

The courts agree that a minor cannot disaffirm where the minor has misrepresented his age.

Promissory estoppel is a contractual doctrine that includes the following considerations:

The courts use the doctrine of promissory estoppel to enforce noncontractual promises.; Under this doctrine the promisor reasonably expects that the promisee, in reliance on the promise, will be induced by the nature of the promise to take action or refrain from taking action and the promisee does so.;

D owes C an undisputed debt of $1,000. D writes a check to C for $700, and writes, "paid in full" on the check. C cashes the check.

The creditor can still collect the $300 because there was no consideration given by the debtor for C's promise to accept the amount as paid in full.

Joe goes to the dentist to have a tooth extracted. Joe never signs a written contract for this service, and he and the dentist never made an oral agreement either. Later the dentist bills Joe, and Joe refuses to pay. After that, the dentist sues Joe. Which of the following is true?

The dentist can recover under quasi-contract.; The dentist can recover under and implied contract theory.;

Harold Homeowner offers to sell Bill Buyer his house for $95,000 and agrees to keep the offer open for 10 days. On the third day, the house burns down.

The destruction of the house brings the offer to an end.

Sam Patten owns a very successful shoe shop known as the Progressive Shoe Store. Arlen Coyle desires a franchise in order to open a similar store. Mr. Coyle contacts Mr. Patten and is told, "If you will buy Lot 21 in Section 18 and build a building according to the specifications and plans that I provide, I will grant you a Progressive Shoe Store franchise." Mr. Coyle performs these tasks and requests his franchise. Patten refuses to grant the franchise and contends there was no consideration and Coyle cannot hold him liable. Which of the following is most correct?

The doctrine of promissory estoppel applies.

Andrew owns a store in Duval County. His trade extends throughout Jacksonville, but not beyond the county limits. He sells his store to Betty and as part of the transaction agrees not engage in the same business anywhere in Jacksonville for a period of five years.

The geographic restraint is reasonable

Which of the following is correct regarding the contractual liability of incompetent persons?

The incompetent must restore the competent party to the status quo before the transaction by a return of the consideration received or its equivalent in money.

Bill Businessman obtains an exclusive franchise to sell widgets for the Acme Widget Company. The exclusive franchise covers the entire State of Wisconsin for a period of three years.

The manufacturer is obligated to use his best efforts to supply the goods even if no such clause appears in the written franchise agreement.

Sarah Citizen hears about a reward being offered by the local television station for information leading to the arrest and conviction of a local rapist. She supplies the requested information and the suspect is then arrested and convicted.

The offer by the television station was an offer made to the general public to enter into a unilateral contract, which offer Sarah has accepted.

Destruction of the subject matter has what effect on the offer?

The offer is terminated.

Which of the following will not be sufficient to satisfy the consideration requirement for a contract?

The offeree expends both time and money in studying and analyzing the offer.

William agrees to drill a well up to 200 feet deep for John's rural cabin. The contract price is $3,000. After drilling 100 feet, William strikes solid granite rock. He talks to John and explains that this is highly unusual for the area and could not have been anticipated at the time of entering into the contract. He offers to get a special drill, but says it will cost him more money, so that he will be unable to complete the project for the agreed upon price. Because John is anxious to have the well, he agrees to pay William an additional $1,000 to complete the job. However, once the well is finished, he changes his mind and now says he will pay only the originally agreed upon amount.

The parties have agreed to a substitute contract which discharges the original contract. John is obligated to pay the additional $1,000.

Which of the following is correct with regard to an enforceable restraint on trade?

The restraint should be no more extensive than is required to protect a specified business interest.; Restraints typically arise in connection with the sale of a business.; A typical restraint is a covenant not to compete.; All of the above;

Which of the following contracts is covered by Article 2 of the Uniform Commercial Code?

The sale of a new Ford Mustang

Which of the following contracts is covered by Article 2 of the Uniform Commercial Code?

The sale of a new car

Which of the following agreements is unenforceable because of indefiniteness?

The seller agrees to supply a quantity of pears dependent upon the buyer's wish.

A state statute establishes a maximum interest rate of 10% for loans or forbearance of money when the debtor is not a corporation. Arrears and Buck, a large retailer, effectively charges consumers an annual rate of 18% on the unpaid balances of their purchases.

The time-price doctrine may apply.

Which of the following is true about situations where a minor or former minor tries to disaffirm a completed contract in which the consideration furnished to the minor has been lost, stolen, or dissipated?

The traditional rule here is that the minor cannot disaffirm because the minor has nothing to give back to the other party.; The traditional rule here is that the minor cannot disaffirm because the contract is completed, and only uncompleted contracts can be disaffirmed.; Today, the courts all agree that a minor can disaffirm here - without giving anything back to the adult.; Answer = None of the above.

If an attorney prepares a will for you, but has not passed the Florida Bar exam, which of the following is correct? Assume the will meets all the requirements of a will in Florida and the attorney sends you a reasonable bill.

The will is valid, you do NOT have to pay the attorney.

Theodore agreed to pay Hal Hays, the owner of a grocery store, $50 if Hays would send a $100 gift certificate, identifying Theodore as the donor, to a recipient chosen randomly from a phone book. Hays mailed the gift certificate. Which of the following is correct?

Theodore's promise to pay is supported by consideration.

Jack has been in the business of selling carpeting for 20 years. He calls Bob, who is opening another branch of his furniture stores, and offers to sell him 100 yards of carpet at $20 per yard. Bob agrees and sends back the following letter confirming the deal: Dear Jack: As we discussed on the phone January 3, we accept your offer of 100 yards of carpeting at the rate of $20 per yard. We also reserve the right to purchase any additional yardage we need to carpet our other showroom facilities at the same rate for one year from that date. Very truly yours, Bob Which of the following is true?

There is a contract for only 100 yards of carpeting.;

On May 1, X writes Y a letter offering to pay Y $1000 if Y will promise to paint X's house. The offer says nothing about stipulated or authorized means of acceptance. On May 2, Y sends X a letter rejecting X's offer. On May 3, Y changes his mind and (saying nothing about his May 2 letter) telephones X to accept the offer. On May 4, X goes completely insane. On May 5, X receives Y's rejection letter. Which of the following is true?

There is a contract here.

Watts offered to buy a car from Stills for $1,200, in writing, by mail. Stills, thinking that was too low, wrote, "Your bid is low. I will only sell it for $1,400," and mailed the letter to Watts. A few hours later, Stills reconsidered and telegraphed, "I accept your deal, regards, Stills."

There is a contract if the acceptance gets to Watts first.

A orally offers to sell B 100 premium-grade fountain pens but neglects to state the price. B orally accepts. Immediately thereafter, A tries to back out of the deal. At this point in time, which of the following is most likely to be true?

There is a contract, if the parties intended to make a contract and there is a reasonably certain basis for determining the price.

A makes B an offer for the sale of land. The offer says that B "must accept by registered letter." B accepts the offer with a regular letter. Which of the following is true?

There is no acceptance and no contract

Barbara Baker, a wealthy widow, promises the pastor of her church that she will donate $10,000 to the church to help pay off its mortgage, if the stewardship committee can obtain enough pledges for the balance of the $30,000 mortgage. Other pledges are obtained to pay off the mortgage, but now Barbara has changed her mind and plans to take an around the world cruise instead.

There is no consideration for Barbara's promise to pay $10,000.; Barbara's promise is enforceable.;

Chuck, a farmer, has 20 breeding cows. He writes to Pete offering to sell "one of my best breeding cows for $4,000." The cattle are of a variety of breeds. Chuck certainly did not intend to include his number one breeder, El Champeen, worth around $60,000. Pete writes back stating, "accept your offer and will be over this Saturday to pick up my choice, El Champeen."

There is no contract because there is no mutual assent.

A standard-form offer for the sale of goods says that all disputes under the contract must be subjected to arbitration. The standard-form acceptance rejects arbitration. Which of the following is most true? Assume that the acceptance is a definite and seasonable expression of acceptance.

There is no contract if the offeree's acceptance was expressly conditional on the offeror's rejection of arbitration, and the offeror did not agree to reject arbitration.

In addition to the four basic requirements of a contract, which of the following must also occur in order to have a valid contract?

There must be an absence of invalidating conduct and duress.

Assume an offeree mails an acceptance on November 1. This acceptance arrives at the offeror's place of business on November 6. In the meantime, on November 4, the offeree sends the offeror a rejection which arrives on November 5. Which of the following statements correctly describes the situation?

There was a contract on November 1.

Assume an offeree mails a rejection to the offeror on November 1. This rejection arrives at the offeror's place of business on November 5. In the meantime, on November 4, the offeree sends the offeror an acceptance that arrives November 6. Which of the following statements correctly describes the situation?

There was no contract

Which of the following statements is most correct concerning promise to make a gift to a charity?

They are generally enforceable.

Sarah Citizen is working hard on the mayoral campaign of Timothy Talker. She thinks that just a few more votes could win the election, so she promises to pay her friend Violet $50 to register and vote. Violet does so, but Timothy loses the election, and Sarah now refuses to pay.

This agreement is enforceable.

Sarah Citizen is working hard on the mayoral campaign of Timothy Talker. She thinks that just a few more votes could win the election, so she promises to pay her friend Violet $50 to register and vote for Timothy Talker. Violet does so, but Timothy loses the election, and Sarah now refuses to pay.

This agreement is unenforceable and opposed to public policy.

Anna is 88 years old and under the legal guardianship of her daughter. One day Anna receives a telephone call from a health insurance salesman and purchases a $400 a month Medicare-gap insurance policy.

This contract is void.

Marilyn contracted with Bravo Builders to build an addition to her house for $15,000. After digging the foundation, Bravo decides that it will take more work and more concrete than it had originally thought and that it will need to charge an additional $5,000 for the job. Assuming Marilyn agrees, which of the following is correct?

This is a modification of a pre-existing contract, which under common law must be supported by additional consideration on the part of Bravo Builders.

Elmer promises to pay Fred $100 if Fred will register and vote in the next election.

This is a valid contract.

Carl Contractor and Ron Roadbuilder are both engaged in road construction work. They know that several jobs are going to be up for public bids, and agree between themselves that Carl will bid on one job and Ron will bid on the other, so that they both have work for the summer. When the bids are opened, Carl realizes that Ron has bid on both jobs. Ron is awarded both contracts. Carl now wants to sue Ron for breach of contract.

This is an agreement in violation of public policy that will not be enforced by the courts.

Cheryl Cheerleader, a 16-year-old high school junior, orders a new dress to wear to the high school prom. She agrees to pay $200 when the dress arrives. Before the dress comes in, she has a fight with her boyfriend and calls the store to cancel the order.

This is an executory contract.; The dress is a necessary item.;

Elmer promises to pay Fred $100 if Fred will register and vote Republican in the next election.

This is an illegal agreement.

Bill Businessman bets his friend Al Attorney $100 that the Jaguars will win the next Super Bowl.

This is an illegal wagering agreement.

James offers to sell four acres of land to Jennifer for $8,000 and further offers to keep the offer open for one month if Jennifer will pay him $100 for the privilege. Jennifer pays James $100. Which statement describes the payment of $100?

This is an option contract.

The Code provides that a merchant is bound to keep an offer open for a stated period not in excess of

Three Months

A contract that is considered to be "within" the Statute of Frauds must comply with the requirements of the Statute in order to be enforceable.

True

A moral obligation is generally not deemed to be consideration.

True

An incompetent person who lacks understanding of a contract and its consequences can avoid it even if the other party has no reason to know of the incompetent's mental condition.

True

As a general rule, disaffirmance must come either during a minor's minority or within a reasonable time after he reaches majority.

True

Courts treat contracts of incompetents and intoxicated persons essentially the same, except they are stricter with intoxicated persons because of its voluntary nature.

True

Except for gifts, as a general rule offers may be revoked as long as the revocation occurs before the acceptance.

True

If Marie, a minor, disaffirms the purchase of her new car that she has recently wrecked, she need only return the damaged vehicle to validly disaffirm the contract.

True

If a person is unable to understand the nature and effect of entering into a contract, he or she can avoid it.

True

If a person lacks mental capacity at the time they enter into a contract, he/she may avoid liability under the agreement.

True

In a bilateral contract, if one party is not bound, neither party is bound.

True

Ratification of a contract, once effected, is final and cannot be withdrawn.

True

The death or insanity of the offeror or offeree does not terminate an option.

True

The parties may enter into a contract for the sale of goods even though they have reached no agreement on that price.

True

The performance of a pre-existing contractual duty is legally insufficient consideration.

True

Under the common law, in order to be enforceable, a modification of an existing contract must be supported by mutual consideration.

True

A friend of Larry T. sent in Larry's name in response to an advertisement for "10 records for one cent and a promise to buy 6 more over the year." When Larry T. received the records he opened and used them. He knew he hadn't ordered the records.

Under quasi-contract, Larry T. must pay since he knew it was a mistake.

Claudia sells her highly successful hair salon to Carl. In the sales contract, Claudia agrees never to open a hair salon in the state. Which of the following best describes this contract clause?

Unenforceable

Which of the following is untrue regarding the contracts of incompetent persons?

Unlike a minor, an incompetent person can never ratify a contract.

When does acceptance of an offer to enter into a unilateral contract occur?

Upon full performance by the offeree

When may a person disaffirm a contract for the sale of real property?

Upon reaching the age of 18

Which of the following statements is correct concerning usury laws?

Usury laws are intended to prevent the charging of an excessive interest rate.

Vantage telephoned Breyer on December 18, and offered to sell a plot of land to Breyer for $5,000. Vantage promised to keep the offer open until December 27. Breyer said he was interested in the land but wanted to inspect it before making any commitment. Which of the following best describes the legal significance of these events?

Vantage may revoke the offer at will.

Tim, who is a minor, enters into a contract with Violet, who is an adult. Which of the following is correct?

Violet may not disaffirm the contract.

Victor thought that Wade's house was ugly, so he decided to paint it while Wade was out of town. When Wade returned, Victor presented him with a bill for $1,200, the amount a professional house painter would have charge. A court would find:

Wade has no obligation to pay Victor.

Which of the following does not require consideration to be effective and binding?

Waiver of a breach

A and B exchanged standard forms regarding the sale of goods, and the standard forms do not agree on material terms. Under UCC section 2-207, when will there be a contract anyway? Assume that B did not make its acceptance conditional on A's assent to any additional or different terms.

When B's form is a definite and seasonable expression of acceptance.; When there is conduct by both parties recognizing the existence of a contract.:

An acceptance is generally effective at what point?

When dispatched

Under which of the following circumstances would a court be unlikely to enforce an illegal contract?

Where the agreement is with an unlicensed attorney

In which of the following situations does the common law "pre-existing duty" rule not apply?

Where the alleged promise to perform a pre-existing duty really contains some new obligation, no matter how small.; Where unforeseen and unforeseeable circumstances make performance of the pre-existing obligation far more difficult than originally believed.; Where the parties mutually agree to terminate the first contract between them.;

To which of the following situations does the common law "pre-existing duty" rule not apply?

Where the alleged promise to perform a pre-existing duty really contains some new obligation, no matter how small; Where unforeseen and unforeseeable circumstances make performance of the pre-existing obligation far more difficult than originally believed; Where the parties mutually agree to terminate the first contract between them; All of the above;

In which of the following circumstances is a debt settlement not a binding contract?

Where the amount of the debt is uncertain and undisputed

In which of the following situations would a minor be unable to disaffirm a contract which he had made?

Where the minor wishes to perform part of a contract and disaffirm another part of the same contract

Which of the following is not a basic requirement of a contract?

Writing

On May 1, Melforth Realty Company offered to sell Greenacre to Dallas, Inc. for $1,000,000. The offer was made by telegraph and stated that the offer would expire on May 15. Dallas decided to purchase the property and sent a registered letter to Melforth on May 10 accepting the offer. As a result of unexplained delays in the postal service, the letter was not received by Melforth until May 22. Melforth wishes to sell Greenacre to another buyer who is offering $1,200,000 for the tract of land. Has a contract resulted between Melforth and Dallas?

Yes

Aunt Ellie promises her 21-year-old nephew, Robbie, that she will pay him $100 if quits smoking for a month. If Robbie does, is there a binding contract?

Yes, because Robbie gave up a legal right.

Sam owes $5,000 to the First National Bank for a student loan which will come due on January 1 next year. He has been offered a two-year graduate fellowship, but he will not be able to pay the loan back if he accepts the fellowship. The bank manager tells Sam that if he pays $3,000 now, they will forgive the loan. Should Sam accept the offer?

Yes, because the early payment of the loan is consideration that makes the bank's promise binding.

Don has an employment contract with Dunkirk Ice Cream. He sells ice cream and novelty ice cream products. He has nine children and doesn't make enough money, so decides to see if Sealtest will hire him, too. "After all," he reasons, "most stores carry four or five different brands." His employment contract prohibits him from competing. If Don sells for Sealtest, too, will he be in trouble under his contract?

Yes, it is likely to be enforceable during employment.

Arthur mails an offer to Brian on June 15. Brian receives the offer on June 16. Arthur mails a revocation of the offer on June 17. Brian mails a letter of acceptance on June 18. Brian receives the revocation on June 19. Was a contract formed?

Yes, on June 18.

Wes, who is an art collector, offered to buy a Miro original from Le Monde Gallery. Le Monde balked at the price, sending Wes a letter of rejection. The same day it was discovered that the print was not as highly valued as originally thought. Le Monde immediately telephoned Wes to accept his offer. Is there a contract?

Yes, since the acceptance was received before the rejection.

Jack moved from New Hampshire to Florida and decided to have an air conditioner installed in his car. After it was installed, Jack received a bill for $1,200. Jack called the dealer and told him he'd never heard of this service costing more than $500. They argued, but the dealer finally agreed to take $900. Is the agreement enforceable?

Yes, there is consideration for the modified amount.

For there to be consideration for a contract, there must be

a bargained-for detriment to the promisor(ee) or a benefit to the promisee(or).

Donna told Jim that she needed someone to feed her cat while she was on vacation. Jim said to Donna, "I'll feed your cat, if you agree to pay me $30." Donna said to Jim, "I'll do it." At that point, Jim and Donna have:

a bilateral, executory contract.

A contract wherein the obligation to perform that contract is based upon the happening of a specified event, even if the specified event may never occur, is an example of

a conditional contract.

An option is

a contract

On April 2, Jet Co. wrote Ard, offering to buy Ard's building for $350,000. The offer contained all of the essential terms to form a binding contract and was duly signed by Jet's president. It further provided that the offer would remain open until May 30 and an acceptance would not be effective until received by Jet. On April 10, Ard accepted Jet's offer by mail. The acceptance was received by Jet on April 14. For this item only, assume that on April 13 Ard sent a telegram to Jet withdrawing the acceptance and rejecting Jet's offer and that Jet received the telegram on April 15. Under the circumstances,

a contract was formed on April 14.

The following conversation took place between Mary and Ed: "Ed, if you wanted to sell your table, what would you ask for it?" Ed: "I suppose $400 would be a fair price." Mary: "I'll take it, if you will have it refinished." Ed: "Sold." Thus, for that price

a contract was formed when Ed said: "Sold."

Lynn offers to sell his house to Dennis for $50,000. Dennis responds, "I will pay you $50,000 if you will paint the second floor." This response could best be described as:

a counteroffer.

Bingo is

a form of gambling

If the obligor desires to be discharged from his duty, he will have to seek which of the following from the obligee?

a novation

According to common law, informing someone of an intention to do an act or an intention to refrain from acting in a specified manner is considered to be:

a promise

A condition precedent is:

a requirement which must be met before the obligations of the contract are enforceable

An agreement of a seller to supply a buyer with all of the buyer's requirements for certain goods is known as:

a requirements contract.

Sarah offers to pay Allison $150 if Allison will paint her apartment while she is out of town on vacation for two weeks. Allison makes no promise but tells Sarah that she will think about it. While Sarah is out of town, Allison paints the apartment. This is best described as:

a unilateral contract.

George has been declared incompetent by the court and is under the care of his sister. Unknown to his sister, George rents the 30,000-seat civic center for his birthday party. George's contract to rent the civic center is best described as:

a void contract

When the auctioneer brings the hammer down on the bidding, the result is

acceptance

Which of the following is not effective when received?

acceptance

Jackson paid Brady $100 for a 90-day option to purchase Brady's 160-acre farm for $32,000. The option agreement was in writing and signed by both parties. The agreement referred only to the option, its period, a legal description of the farm, and the purchase price. Jackson wrote Brady 30 days later: "I hereby exercise my option to purchase your farm for $32,000 subject to closing details to be worked out by you and my attorney." Jackson's letter

accepts Brady's offer leaving customary details to be worked out during formalization of the contract.

The courts usually consider all of the following matters relevant to contracts except:

adequacy of consideration.

A contract that is offered on a "take-it-or-leave-it" basis is known as a/an:

adhesion contract

On September 10, Harris, Inc., a new car dealer, placed a newspaper advertisement stating that Harris would sell 10 cars at its showroom for a special discount only on September 12, 13, and 14. On September 12, King called Harris and expressed an interest in buying one of the advertised cars. King was told that five of the cars had been sold and to come to the showroom as soon as possible. On September 13, Harris made a televised announcement that the sale would end at 10:00 p.m. that night. King went to Harris's showroom on September 14 and demanded the right to buy a car at the special discount. Harris had sold the 10 cars and refused King's demand. King sued Harris for breach of contract. Harris's best defense to King's suit would be that Harris's

advertisement was not an offer.

All of the following are essentials of an offer to contract except

all terms of the contract must be included in the offer.

A major distributor of popular sportswear offers a franchise agreement to retail stores. The contract is prepared on a standard form and offers terms on a take-it-or-leave-it basis. Such a contract is called:

an adhesion contract

The elements of a contract include all of the following except:

an illegal purpose.

Alice says to Brian, "If I decide to buy a word processor next year, I will buy it from you." This is an example of:

an illusory promise

Bob picks up a bag of pretzels off the counter at the hot dog stand, opens it up and begins eating pretzels. He has made:

an implied contract to pay for the pretzels.

When a client accepts the services of an accountant without an agreement concerning payment, the result is

an implied-in-fact contract.

The concept of consideration in contract law includes:

an inducement to each party to make a return exchange.

All of the following EXCEPT which one will terminate a contract offer?

an inquiry by the offeree as to size

A definite proposal requesting a forbearance and made by one person to another is

an offer

At an auction sale, each bid is:

an offer

Gail sent a letter of acceptance to an offer that has expired. Gail has made:

an offer

A contract that binds the offeror to keep an offer open for a specified period of time is known as:

an option.

A promise by the Rive Gauche restaurant to buy all the oil it wants this winter to heat its business from Ollie's Oil Corp. would be:

an unenforceable, illusory promise.

Where the amount of the debt to be paid has not been agreed upon by the parties, there is

an unliquidated debt.

In contracts governed by the UCC, an authorized means of communication is:

any medium that is reasonable under the circumstances.

Express contracts and implied in fact contracts.

are both genuine contracts.; are equally enforceable.;

Bill sends Carla an offer by express mail. Carla receives it at 10 a.m. on Tuesday. At 11 a.m. on Tuesday, Carla delivers an acceptance to Post Office express mail, but due to an error, the letter is not sent out by the Post Office until Wednesday at 8 a.m. At what time does the law consider the acceptance to be effective?

at 11 a.m. on Tuesday

Opal offered, in writing, to sell Larkin a parcel of land for $300,000. If Opal dies, the offer will

automatically terminate prior to Larkin's acceptance.

A statute or court decision that makes an offer illegal will

automatically terminate the offer

Egan, a minor, purchased Baker's used computer for Egan's personal use. Egan paid $200 down on delivery and was to pay $200 thirty days later. Twenty days later, the computer was damaged seriously as a result of Egan's negligence. Five days after the damage occurred and one day after Egan reached the age of majority, Egan attempted to disaffirm the contract with Baker. Egan will

be able to disaffirm despite the fact that Egan was not a minor at the time of disaffirmance.

An offer need not take any particular form to have legal validity. To be effective, however, it must:

be communicated to the offeree.

For an offer to confer the power to form a contract by acceptance, it must have all of the following elements except

be communicated ~ to the offeree by the offeror.

In order to satisfy the consideration requirement to form a contract, the consideration exchanged by the parties must

be legally sufficient

The person who makes an offer in an auction is the

bidder.

A contract in which both parties exchange promises is a:

bilateral contract

To constitute consideration, a promise must be:

binding

Dad promises to pay Junior $100,000, in exchange for Junior's promise to refrain from smoking until he reaches the age of 21. This agreement is:

binding, if all the other elements of a contract are present, because there is consideration here.

An offer to sell a tract of real property is terminated at the time the

buyer learns of the sale of the property to a third party.

A newspaper advertisement made to the general public:

can be revoked by using a similar newspaper advertisement.

Ratification

can only be of the whole and not a part of the contract.

A contract made by a mentally impaired person; who has not been adjudicated incompetent:

can only be ratified after the person has regained his mental capacity.

After a person reaches majority and seeks to ratify a contract entered into as a minor, he/she

can ratify the contract only as a whole.

When a minor ratifies a contract, the minor:

can ratify the contract only as a whole.

When a person who just became an adult ratifies a contract made as a minor, he

can ratify the contract only as a whole.

In general, contracts made by incompetent persons:

can sometimes be ratified.

Contracts made by severely intoxicated people:

cannot be ratified until the person has regained his mental faculties.

Exculpatory clauses:

cannot relieve a party from liability for battery.

The UCC Battle of the Forms rule:

changes the mirror image rule.; may yield different results depending on whether the parties are merchants.; may result in formation of a contract quite different from the terms of the offer.;

The main ingredient which causes an option to be a binding offer is

consideration.

The requirement that each party to a contract must intentionally exchange something of value as an inducement to the other party to make a return exchange is known as:

consideration.

Bill and Mary are merchants. Bill orders 100 computers from Mary which she accepts. In the order he specifies that he wants model ABC. Mary, a long-time friend to Bill, knows what the computers will be used for and substitutes Model XYZ, computer with similar capabilities, but not exactly the same. If Bill did not limit acceptance to the original offer, and did not object to the substitution,

contract formed for Model XYZ.

Which of the following is ordinarily competent to give binding assent to a contract?

convicts

Acceptance of an offer by the offeree after the time for acceptance has expired:

could be a new offer

William recently sold his successful business to Janice. The contract for the sale contained an unreasonable restriction that did not allow William to open a similar business for 15 years. The courts would, in this instance,

delete the unreasonable portions of the contract.

A letter of acceptance, in answer to an offer made by letter, is effective when

deposited in the mail.

Melissa mailed an offer to Jerry to sell him 8 ounces of gold at $450 per ounce. The letter arrived on May 1, and that afternoon, Jerry sent a letter of rejection to Melissa. The next morning, Jerry learned that the price of gold had gone to $490 per ounce, so he immediately mailed a letter of acceptance to Melissa and asked her to ignore the letter of rejection. The rejection was received by Melissa on May 3, and the acceptance was received on May 4. A court would find that a contract between Melissa and Jerry:

did not come into existence, because the rejection was received by Melissa prior to the receipt of the acceptance.

The exercise of the power to avoid a contract is known as ____________.

disaffirmance

Unless the offer states otherwise or a rejection has been previously sent, acceptance is generally effective when

dispatched by the offeree.

The effect of the making of a partial payment to satisfy a debt which is certain in amount and which is past due is an example of the rule that:

doing what one is bound to do is not consideration.

A telephoned acceptance is:

effective when and where acceptance is spoken into the phone.

The Uniform Commercial Code does not apply to:

employment contracts.; service contracts.; insurance contracts.; contracts involving real property.;

Mork is an alien who resides in this country. If Mork makes a contract with a United States citizen, the contract is

enforceable

A written promise by a shipbuilder to an iron ore producer "to purchase all the iron ore I may need during the next year" is

enforceable if the producer had agreed to sell

An agreement to refrain from a particular trade or business is

enforceable, if the purpose is to serve a legitimate business purpose.

Promises to make gifts to charities are generally

enforceable.

A contract which contains a clause which excuses one party from liability for his/her negligent conduct is

exculpatory

A contract that has been fully performed by all of the parties is a(n)

executed contract

A contract under which one or both parties have not as yet fully performed is termed a(n):

executory contract

On the first day of the month, Thomas and Moore orally agreed that Thomas was to deliver to Moore's place of business a case of fruit on each Monday of the current month. Moore was to pay the $100 price on the first day of the following month. On the 15th of the month, the agreement should be classified as

executory.

Unconscionability:

exists both at common law and under Article 2 of the UCC.

A(n) ___________ contract is one in which the terms have been definitely and specifically stated and agreed upon.

express

An offer under Article 2 of the UCC that is irrevocable even though no consideration is given to keep it open is known as a/an:

firm offer.

The Uniform Commercial Code makes certain offers irrevocable even though no consideration is given to keep the offer open. Such offers are known as:

firm offers

An offer which does not specify when it will terminate remains open:

for a reasonable period of time, depending on the circumstances

If no termination date is specified for an offer, the offer will remain open:

for a reasonable time.

Consideration that involves the promise to refrain from doing an act is called:

forbearance

Acceptance of a unilateral offer occurs upon:

full performance by the offeree.

A raffle is

gambling

Able Sofa, Inc. sent Noll a letter offering to sell Noll a custom-made sofa for $5,000. Noll immediately mailed a letter to Able purporting to accept the offer. However, the post office erroneously delivered the letter to Abel Soda, Inc. Three days later, Able mailed a letter of revocation to Noll that was received by Noll. Able refused to sell Noll the sofa. Noll sued Able for breach of contract. Able

has a binding contract with Noll.

The attempted acceptance of an offer by a person to whom the offer was not made:

has the effect of an offer.

Abel owes Baker $100. Baker, tired of chasing Abel and listening to his excuses, finally corners Abel one day after class and tells him that if he pays him $50, Baker will cancel the entire debt. Abel quickly agrees, pulls out a wad of money, peels off a $100 dollar bill, demands and receives $50 in change. That night as Baker mulls over the situation, he gets more and more upset. When, the next day, Baker demands the other $50 from Abel:

he is legally entitled to it.

A minor, M, has obtained necessaries on credit. It is correct to say that

he is only liable for the reasonable value of the necessaries.

Theresa is a travel agent at the Fly Away Travel Agency. She has signed an agreement with her employer which prohibits her from working in any like business in two towns within a 60 mile radius of where she works. If she wants to quit her job and go to work for another travel agency, it is likely that:

if no trade secrets are involved, and she has no dominion over customers, a court would rule the restrictions to be invalid.

Last year, a series of arsons occurred in the City of Swelter. Early this year, Swelter's City Council adopted this resolution: The City will pay $10,000 for the arrest and conviction of anyone guilty of any of the arsons committed here last year. The resolution was telecast by the city's sole television station once daily for 1 week. Immediately thereafter the local television station ceased operations. The city's offer will terminate

if the City Council by resolution repeals its reward offer and causes this resolution to be broadcast once daily for a week over two local radio stations.

A promise to purchase such quantity of goods as one may desire is a(n)

illusory promise.

Rail, who was 16 years old, purchased an $800 computer from Elco Electronics. Rail was unhappy with the computer. Two days after reaching the age of 18, Rail was still frustrated with the computer's reliability. He returned it to Elco, demanding an $800 refund. Elco refused, claiming that Rail no longer had a right to disaffirm the contract. Elco's refusal is

incorrect, because Rail disaffirmed the contract within a reasonable period of time after reaching the age of 18.

Twentieth century contract law is generally characterized by:

increased government intervention into private contracts.

An ad in a newspaper or a circular describing goods and stating prices would generally be considered a(n):

invitation to buyers to make an offer to buy goods.

If not an offer, the first statement made by one of two persons is most properly termed a(n):

invitation to negotiate.

Usually, advertisements are construed by the courts as:

invitations to negotiate.

A covenant not to compete is valid in Florida if it

is 6 months or less in time.

X and Y contract for X to build Y a house for $150,000. Later, without terminating the first contract, X and Y modify that contract so that Y will pay X $175,000. In return, X promises to build exactly the same house, but one day earlier than in the previous contract. Assume that all the other elements of a contract besides consideration are present; this is a consideration question. This second contract:

is a binding contract.

A "grumbling acceptance":

is a valid acceptance if it meets all other tests for a good acceptance.

An offer for a unilateral contract:

is accepted by full performance of the requested act.

A and B make a bet concerning whether a certain rich citizen will die within the next year. Neither party has any economic interest in this person's fate, except for that created by the bet. This agreement:

is an unenforceable wagering agreement.

If the offeree dies before the offer has been accepted, the offer:

is automatically revoked by the death of the offeree.

Employer promises to pay Retiree a $10,000 per year pension for the remainder of Retiree's life in return for Retiree's promise to pay Employer $1 per year. Retiree foreseeably relies on the promise by taking out a mortgage on a small vacation home. Later Employer, who never intended to pay the pension, unfairly reneges on his promise. Employer's promise:

is binding under these circumstances.

X and Y have a contract which obligated X to sell Y 100 boxes of screws for $100. The parties orally modify the contract so that X will sell Y the same 100 boxes of screws for $125. The second agreement:

is binding.

Andrew agrees to paint Betty's house for $500. Two days after he starts the job, he decides that As a general rule, a written promise to pay a debt barred by the statute of limitations:

is enforceable

When Western Chicken Ranch offered to sell eggs to Deb's Dinner at 35 cents per dozen, Deb said, "I promise to pay for as many eggs as I order in the next six months." Deb's promise:

is illusory and therefore unenforceable.

An implied contract is a contract that:

is inferred or deduced from the conduct of the parties.

Smith states, "I am getting tired of trying to keep my collection of Playboys and National Geographics in order, and if I get a good offer to sell them, I will take it immediately." Bright, overhearing the statement immediately responds, "I make you a good offer of $400." This transaction

is not a contract because there was no offer and acceptance.

Past consideration

is not adequate consideration.

An acceptance that materially deviates from the terms of the offer:

is not an acceptance but a rejection.

Generally, a promise to make a gift:

is not enforceable.

A person who is a member of a group to whom an offer is made and finds the lost item

is not entitled to the reward, unless she knew of the reward before finding the item.

Normally, an illegal contract:

is unenforceable.

When a person enters into a written agreement while drinking alcohol or using drugs, the agreement

is voidable if the party didn't know the general nature of the agreement due to the alcohol or drugs.

A contract is considered to be void, if:

it is missing an essential element of contract

A nominal consideration is usually not consideration because:

it suggests the absence of a genuine bargain.

Payne entered into a written agreement to sell a parcel of land to Stevens. At the time the agreement was executed, Payne had consumed alcoholic beverages. Payne's ability to understand the nature and terms of the contract was not impaired. Stevens did not believe that Payne was intoxicated. The contract is

legally binding on both parties.

In determining whether the consideration requirement to form a contract has been satisfied, the consideration exchanged by the parties to the contract must be

legally sufficient.

If a tort and a contract are so intertwined that to enforce the tort, the court must enforce the contract, the minor is not liable in tort. This is the definition of:

liability for tort connected with contract.

Divided Parcel Service includes the following on its mailing receipts: "We are not responsible for any damages to packages whether or not through the fault or negligence of our employees. Send packages at your own risk." Mary reads this clause but sends her watch back to Bulova Co. to be repaired anyway. The watch is destroyed which the DPS driver uses the package for a ball and tosses it to his buddy. Mary is

likely to collect from DPS since exculpatory clauses generally violate public policy.

The statute of limitations

limits the time period for a creditor to sue a debtor on a debt.

A debt in which the amount owing is not disputed and can be expressed as a certain sum of money is known as a(n)

liquidated debt

An item considered a necessary is

lodging; medicine; food;

Toni Minor purchased an automobile from Majority Motors on September 17. She attained the age of majority on October 5. In the early morning of October 6, she was involved in an accident that destroyed the vehicle, which was not insured. On November 1, she informed Majority Motors that she would make no further payments. If Majority Motors sues, it would

lose because disaffirmance is permitted even though the property has been destroyed.

A large grocery store advertised a gourmet type of ham for $5.00 a pound. It usually sold for $9.00. P went to the store and demanded 500 pounds of the particular ham. The store refused to sell P any, and P sues. P would

lose; there was no offer; hence, he could not accept

Generally, one who responds to an advertisement by offering to purchase the goods advertised at the advertised price is:

making an offer.

In order to form a contract, the parties must:

manifest their agreement objectively.

A gross disparity in the values exchanged as part of a contract:

may be a significant factor in a determination that a contract is unconscionable.; may be sufficient grounds for denying specific performance.;

A clearly illegal provision in an agreement:

may be divisible from the rest of the agreement, which means that the court will enforce the agreement without it.

The president of Deal Corp. wrote to Boyd, offering to sell the Deal factory for $300,000. The offer was sent by Deal on June 5 and was received by Boyd on June 9. The offer stated that it would remain open until December 20. The offer

may be revoked by Deal any time prior to Boyd's acceptance.

So-called "contracts of adhesion":

may be unconscionable.

Which of the following is required for a "firm offer?"

merchant, in writing, signed, not over 90 days

A "firm offer" under UCC

must be in writing

On April 6, Apple entered into a signed contract with Bean, by which Apple was to sell Bean an antique automobile having a fair market value of $150,000, for $75,000. Apple believed the auto was worth only $75,000. Unknown to either party the auto had been destroyed by fire on April 4. If Bean sues Apple for breach of contract, Apple's best defense is

mutual mistake.

A minor generally is liable for the reasonable value of

necessaries

Before granting an injunction enjoining a former employee from competing in a described territory, the courts insist that the employer demonstrate that the restriction is ___________ to protect the employer's legitimate interest.

necessary

If a minor enters a contract which is interwoven with a tort, will the minor be held liable in tort?

no

The manager of the local J.C. Penney store sends you a signed personal notice that its chainsaws will be on sale for 30% until January 15th. If you go in on January 12th and they refuse to give you 30% off, you will win if you argue that

no consideration is needed for the promise.

If an offeree sends a rejection, then changes her mind and sends an acceptance, a court would hold that:

no contract exists if the rejection is received before the acceptance is received.

Baker Corporation sent a letter to Sampson Company in which Baker offered to purchase 10 acres of certain real estate from Sampson for $4,000. Sampson responded that it would sell 8 of these acres for that price. Baker and Sampson have created

no contract in this connection.

A person who is OLDER than the age of majority may disaffirm a contract if:

none of the above.

Joanne, a minor, sold her laptop computer to Bruce, an adult. Bruce then sold the laptop to Anna, also an adult, who had no knowledge of the fact that the original owner was a minor. Under the Uniform Commercial Code, Joanne may:

not avoid her contract to Bruce and may not recover her laptop.

Ordinarily, a promise to perform an existing legal obligation is:

not consideration.

In general, if ALL of the elements of a contract are not satisfied, the courts will:

not enforce the contract.

Gina promised to pay Bert $500, if Bert would stop stealing the radio from Gina's car. If Bert stops taking the radio, he will be found to have:

not given consideration for Gina's promise to pay.

West, an Indiana real estate broker, misrepresented to Zimmer that West was licensed in Kansas under the Kansas statute that regulates real estate brokers and requires all brokers to be licensed. Zimmer and West mutually agreed that Zimmer would pay a 5% commission for selling Zimmer's home in Kansas. Zimmer signed a written document reflecting such agreement but West did not sign the document. West sold Zimmer's home. If West sued Zimmer for nonpayment of commission, Zimmer would be

not liable to West for any amount because West violated the Kansas licensing requirements.

On Danny's 18th birthday his Uncle Harris promised in writing to pay him $5,000 if Danny would drink no alcoholic beverages for the next two years. Danny drank no alcoholic beverages for the next two years as requested. Uncle Harris refused to pay on Danny's 21st birthday, claiming that Danny actually benefited himself by drinking no alcohol for those two years, thus, there was no consideration. If Danny sues Uncle Harris he will

nothing will happen because there was no consideration. Danny couldn't legally drink and thus Danny had nothing to give up to make the contract enforceable.

When a minor disaffirms a contract, he must

notify the other party and return any property he has from the other party provided he has it.

Alice attends an auction during which she makes a bid on a painting. Alice's bid is a(n):

offer

As a general rule, the acceptance must be the mirror image of the ____________.

offer

If the specific subject matter of an offer is destroyed before the offer is accepted the:

offer is automatically terminated.

The person to whom an offer is made is called the:

offeree.

An option differs from the ordinary offer in that

offers need not be. options do not terminate upon death of the offeree; offers do.;; options do not terminate upon insanity; offers do.;; options can't be terminated by revocation; offers can.;;

A person may ratify a contract

only after his 18th birthday.

If an offer of a reward for finding a lost object is made to the general public, it may be accepted

only by an individual who knew of the reward.

An offer may be accepted

only by the designated offeree.

Arthur offers to sell some land to Bill and says that he will keep the offer open for two weeks if Bill will pay him for the privilege. Bill pays $10. A/An ____________ contract has been formed.

option

A binding promise to keep an offer open for a stated period of time or until a specified date is called a(n):

option contract

A promise to pay a contractor a bonus to complete construction of the building according to the contract is:

ordinarily not binding on the promisor.

While Jane is swimming she cuts her foot on some coral and is quickly surrounded by several sharks. Within earshot of several startled sunbathers, Jane screams that she will pay one million dollars to anyone who saves her. Gertrude, the lifeguard on duty, throws Jane a line and pulls her in. When Jane refuses to pay, Gertrude sues. If, instead of making her offer while dodging sharks, Jane said nothing until after she was rescued by a jogger, and only then promised to pay the million dollars, the court would rule that:

past consideration is not legally sufficient to support a contract.

The time-price differential is the difference between the:

price charged for cash sales and the price charged for credit or installment sales.

Carson Corp., a retail chain, asked Alto Construction to fix a broken window at one of Carson's stores. Alto offered to make the repairs within 3 days at a price to be agreed on after the work was completed. A contract based on Alto's offer would fail because of indefiniteness as to the

price involved

A person who makes a promise is a(n):

promisor.

A minor may NOT disaffirm a contract for

purchase of food; sale of land;

An obligation imposed by law where there has been no agreement or expression of assent by word or act on the part of either party involved is a(n):

quasi contract.

Denise Smolen hired David Vause to construct an exercise center in her home. After completing the job, he sent her a bill for $3,000. Based on the cost of similar work done for several of her neighbors, Smolen stated that $2,000 was a fair price. Vause said that the market rate for quality work was $3,000, but that he would accept $2,500. Smolen agreed and remitted a $2,500 check in full payment of the debt. The parties have

reached an accord and satisfaction

Article 2 of the UCC does not apply to a sale of:

real estate

A letter of revocation for an existing offer takes effect when:

received by offeree

Which of the following statements is correct?

received by offeree.

Which of the following statements is correct?

received by offeror.

An offer is effective when it is

received by the offeree.

Instead of refusing to enforce an unreasonable covenant, some courts will:

reform the agreement.; uphold reasonable restrictions.; delete unreasonable restrictions.;

Which of the following does not terminate an offer?

refusal to respond by offeree

Once a court finds a contract or a term of the contract to be unconscionable, it may:

refuse to enforce the entire agreement.; enforce the contract without the unconscionable provision.; change the unconscionable term so as to avoid any unconscionable result.;

A statute which prescribes standards for those who seek to practice a profession would be what type measure?

regulatory

A/an ___________ is a measure designed to protect the public from unqualified practitioners.

regulatory license

A counteroffer is a(n):

rejection of the original offer.

The manifestation of an unwillingness to accept an offer is called a

rejection.

An agreement is an essential element of a contract. Ordinarily, the required mutual assent is achieved by means of an offer and an acceptance. Acceptance

requires manifestation of an intent to accept.

Tom Teen, age 16, purchased a radio from an adult. Tom was not asked how old he was. In order to disaffirm the contract, Tom must

return the radio if he has it.

An offer may terminate by

revocation; rejection; death of either party;

Gambling is made up of

risk, reward, consideration.

According to the UCC, an enforceable agreement involving the transfer of title of goods from a seller to a buyer for a price is called a(n):

sale

Contract law is significant in that it is basic to other fields of law such as:

sales of personal property.; commercial paper.; secured transactions.;

An announcement that an auction sale is to be conducted "without reserve," means that the:

seller cannot withdraw the goods if the bids are too low.

Ann, a minor, disaffirmed her agreement to buy $127 worth of cosmetics from Mary Kay Company. She had used up all the eye shadows, lipsticks and powders. The general rule is that:

she may disaffirm but she has to return the makeup that is not used up

If Alice wishes to withdraw her bid after the auctioneer's hammer falls:

she may not do so because her offer has been accepted.

Contracts are governed primarily by:

state common law.

John operates a small repair business and is in desperate need of a certain type of building material. He obtains the material from a large corporation, but is charged a grossly unreasonable price and is forced to buy other material he does not need. In view of the buyer's unequal bargaining power and unreasonable terms of the contract, this may be a case of:

substantive unconscionability.

John Q. Public is insane but has not been adjudged so by a court of law. If Mr. Public enters into a contract,

the contract may be voidable at the option of the insane person, if he lacked the mental capacity to comprehend the subject matter.

Farmer Frances has a long standing relationship with Veronica's Vegetable Market whereby several times a month Frances would drop off a load of produce at the market and return a few days later to pick up her agreed upon percent of the selling price. On one particular delivery day, Veronica notices Frances dropping off her customary load of produce, but no words are exchanged. When Frances returns a week later to pick up her money, the produce is sitting exactly where she had left it, too spoiled to now be sold. When Frances sues Veronica:

the court would find that a contract existed because, under the circumstances, if Veronica did not want the produce she had an obligation to tell Frances.

As a general principle of contract law:

the courts do not evaluate the adequacy of consideration.

In a unilateral contract, the consideration of the promise is:

the doing of the act called for by the promisor.

The validity of a contract is not affected by:

the fact that the contract turned out to be a bad bargain for one of the parties.

Whether the time within which a minor disaffirms a contract constitutes a reasonable time is determined by:

the facts of the case.

When a minor commits a tort while completing a contract and the tort and a contract are so "interwoven" that they cannot be separated, then

the law of contracts will prevail.

When personal property is transferred to a third person who is unaware that it was originally purchased by a minor

the minor cannot get the property back from the third person under the UCC.

In Florida when a minor falsely advises the other party that he is of the age of majority and, based upon the misrepresentation, the other party enters into a contract with the minor:

the minor has lost his right to disaffirm the contract because of the misrepresentation.

When a minor falsely advises the other party that he is of the age of majority and based upon that misrepresentation, the other party enters into a contract with the minor:

the minor has lost his right to disaffirm the contract because of the misrepresentation.

UCC section 2-207 (the Code's "battle of the forms" provision) says that additional terms in the acceptance are part of the contract when both parties are merchants, UNLESS:

the offer expressly limited acceptance to its own terms.; the additional terms materially altered the offer.; the offeror gave notice of his objection to the additional terms within a reasonable time.; all of the above.;

Ordinarily acceptance of an offer is effective at the time the acceptance is dispatched. The rule does not apply if

the offer provides that an acceptance shall not be effective until actually received.

If Kamuela had mailed a revocation the same day Terry telegraphed her acceptance, the court would hold that:

the offer was not revoked since revocations are effective only on receipt, whereas Terry's acceptance was effective when sent.

If, in the example above, before attempting to accept, Nora learns that Nick has sold the bracelet to someone else:

the offer would be revoked.

For an offer to be valid, the following elements must be present:

the offeree must have knowledge of the offer.

In an offer for a unilateral contract, once an offeree has made substantial progress toward rendering the requested performance, most courts would hold that:

the offeror had, at least temporarily, lost the power to revoke.

The subject matter of a contract may relate to:

the performance of personal services.; the construction of a house.; the transfer of ownership of property.;

An offer may be accepted by:

the person to whom it is made.

If the offeror has the power to terminate the offer before acceptance, the offeror has

the power of revocation.

An agreement to refrain from a particular trade or business is enforceable if:

the purpose of the restraint is to protect a property interest of the promisee.; the restraint is no more extensive than is reasonably necessary to protect that interest.;

The plaintiff in a quasi-contract action can recover:

the reasonable value of the benefit conferred upon the defendant.

The effective moment of rejection is

the receipt of the rejection by the offeror.

Sometimes adults who can't sue a minor on the contract will sue in a tort. In order to be successful

the tort must be clearly separate from and independent of the contract.

Natalie's business law instructor offers to sell his Volvo for $100 to the first student to accept his offer. The Business Department's Associate Dean overhears this as she is walking down the hall and sticks her head into the room and says, "I accept." A moment later Natalie jumps up and yells, "I accept." When the court is asked to make a determination, most likely it will find that:

there is a contract between the instructor and Natalie since she was the first student to accept.

Smith stated to Burns that he, Smith, would sell a certain typewriter to Burns for $300. In fact, Smith never intended to sell the typewriter, and this can be proven. Burns agreed to buy it for the $300 price. With regard to these facts

there is a contract if a reasonable person would think that Smith was making an offer, and if Burns thought so.

If an offer requires acceptance by fax and the offeree mails acceptance:

there is a contract if the acceptance is actually received within the time the authorized means would have arrived.; under the Restatement, if the acceptance is received within the time the authorized means would have arrived, the acceptance is effective when sent.;

A orders 100 19-inch color television sets from B, and requests prompt shipment of the goods. B promptly ships A 100 21-inch color TVs. B did not tell A that the shipment of nonconforming TVs was in accommodation. In this case:

there is a good acceptance, but B has breached the contract by shipping nonconforming TVs.

A contract will not come into existence, unless:

there is a valid offer and acceptance.

In deciding whether consideration necessary to form a contract exists, a court must determine whether

there is mutuality of consideration.

Cesare offers to sell a gram of arsenic to Lucrezia for $50. Lucrezia says, "No." However, a moment later she changes her mind and tells Cesare that she accepts his offer:

there is no contract because of Lucrezia's rejection.

Sarah offered to buy a sofa from Mark for $400 and stated, "This offer must be accepted in writing by August 5." On August 2, Mark wrote a letter to Sarah that stated that Mark accepted the offer. Mark signed the letter, but then changed his mind and tore up the letter. The court will find that:

there was no valid contract between Sarah and Mark, because Mark never communicated acceptance to Sarah.

If, in the example above, Nick had decided to revoke his offer but was unable to communicate with Nora because her phone was out of order, and she accepted before he could tell her that he was revoking his offer:

there would be a valid contract.

Adhesion contracts are sometimes held to be unconscionable, but their use is often justified. An adhesion contract is most appropriate if a seller

thereby realizes efficiencies that reduce transaction costs.

Ratification can occur in three ways. They are:

through express language.; as implied from contract.; through failure to make a timely disaffirmance.;

In non-compete agreements the important factors are

time and area

Adolf Anderssen performed accounting services for Carla Jung and sent her a bill for $500. She responded in good faith that the value of the services was $300 but that she was willing to pay $375 to avoid litigation. Accordingly, she sent Anderssen a check for that amount marked "payment in full." Anderssen received the check, crossed out the notation "payment in full," cashed it, and filed suit in small claims court for $125 and costs. If he desired to recover the full $500, Anderssen's best course of action was

to return the check

Which of the following would least likely considered a "necessary" as discussed in class?

transportation

A contract that is deemed to be too harsh or oppressive to one of the contractual parties may be unenforceable under the concept of:

unconscionability

The Uniform Commercial Code provides that a court may scrutinize every contract for the sale of goods to determine whether in its commercial setting, purpose, and effect the contract is:

unconscionable

Contractual capacity of the parties is a prerequisite to the formation of a binding agreement, but the consequences of different forms of incapacity vary. The contracts of a person are void if (s)he is

under guardianship of a court.

An oral contract that is covered by the statute of frauds is:

unenforceable

Tim Carlton was swimming at the beach when he happened to see Fay Hudson struggling in the water. Carlton saved Hudson's life. Hudson was so grateful that she promised Carlton a job for the rest of his life. Carlton went to work for Hudson, but a few months later Hudson found that she did not get along with Carlton and demanded Carlton's resignation. The contract is

unenforceable because Carlton did not give consideration.

Joe says to Sue: "I promise to pay you $1500 if you'll paint my house." Sue begins to paint Joe's house. Right now, she is half-finished. At this point in time, the contract between Joe and Sue is:

unilateral and executory.

A contract which requires an offeree to perform an "act" instead of a promise is a(n)

unilateral contract

Kay, an art collector, promised Hammer, an art student, that if Hammer could obtain certain rare artifacts within 2 weeks, Kay would pay for Hammer's post-graduate education. At considerable effort and expense, Hammer obtained the specified artifacts within the 2-week period. When Hammer requested payment, Kay refused. Kay claimed that there was no consideration for the promise. Hammer would prevail against Kay based on

unilateral contract

A contract that requires a performance from only one of its parties is called a(n):

unilateral contract.

Ted offers to pay Dennis one million dollars, plus one of his television stations, if Dennis can better Ted's record-setting time in the Newport to Bermuda yacht race. Dennis accepts the challenge. This would be an example of:

unilateral contract.

An obligation which is either contested as its existence or as to its amount is considered a ___________ debt.

unliquidated

Harris wrote Douglas a letter which might be construed alternatively as an offer to sell land, an invitation to commence negotiations, or merely an invitation to Douglas to make an offer. Douglas claims that the communication was a bona fide offer which he has unequivocally accepted according to the terms set forth therein. In deciding the dispute in question, the court will

use an objective standard based on how a reasonably prudent businessperson would have interpreted the letter to Douglas.

Offers

use objective standard for intent.

An output and requirement contract is

usually enforceable

An output or requirement contract is

usually enforceable

Silence in response to an offer:

usually is not an acceptance.

A statute establishing a maximum rate of permissible interest is a

usury law

A/an ___________ is a law establishing a maximum rate of permissible interest for which a lender and borrower of money may contract.

usury statute

A contract which requires a buyer to purchase all that he needs from a seller is

valid

In a situation in which there is a unilateral mistake the contract is

valid

Ace Electric Company has entered into an agreement with Tri-State Metal to buy its actual requirements of conduit for six months from Tri-State. Tri-State has agreed to sell all required conduit for the next six months to Ace. The agreement is

valid and enforceable.

Certain contracts have absolutely no effect and are not recognized under law. If two or more parties enter into such an agreement, it is

void

In October, a court holds Rodney Doodle mentally incompetent and appoints a guardian for him. Rodney, a former business law professor, is the victim of a debilitating mental illness brought on by teaching the law of contractual capacity for too many years. In November, Rodney escapes his guardian's care and takes off on his own. While eating lunch with Ed Sleaze, who knows about Rodney's condition, Rodney signs an agreement to sell a valuable property he owns. When he signs the contract, Rodney believes that he is a movie star signing an autography for an adoring female fan. The agreement between Rodney and Ed is best described as:

void

When there is no consideration for a promise, the agreement is:

void

Lee has been declared incompetent by the court and is under the care of his sister. Unknown to his sister, Lee rents the 30,000-seat civic center for his birthday party. Lee's contract to rent the civic center is best described as a:

void contract

A contract whose formation is induced by duress (force or compulsion) is:

voidable

A minor's contract is ordinarily

voidable

Steven makes a material misrepresentation of fact regarding his motorcycle to Thelma who agrees to buy the motorcycle based upon the misrepresentation. This contract is.

voidable

Meed entered into a written agreement to sell a parcel of land to Beel for $80,000. At the time the agreement was executed, Meed had consumed a large amount of alcoholic beverages which significantly impaired Meed's ability to understand the nature and terms of the contract. Beel knew Meed was very intoxicated and that the land had been appraised at $125,000. Meed wishes to avoid the contract. The contract is

voidable at Meed's option.

A contract with an intoxicated person is

voidable by intoxicated person.

Except for necessaries, contracts with minors are generally

voidable only by minor.

Sandy fraudulently induced Fred to enter a contract with her. A court would treat the contract as:

voidable.

Consideration is:

what is demanded by the promisor as the price for the promise.

James offers to sell his fishing boat to Brenda for $3,000. Brenda says she will apply for a loan and will buy the boat within a week. A contract is formed:

when Brenda tells James she will buy the boat.

Susie offers to sell Ralph her Mustang for $500. Ralph says it will take him a week to talk his dad into giving him the money, but that he wants it. A contract is formed:

when Ralph accepts the deal.

In general, a mailed notice of revocation is effective:

when received

Able Sofa, Inc. sent Noll a letter offering to sell Noll a sofa for $5,000. Noll immediately sent a letter to Able accepting the offer. However, the postal service erroneously delivered the letter to Abel Soda, Inc. Three days later, Able mailed a letter of revocation to Noll that was received by Noll. Able refused to sell Noll the sofa. Noll sued Able for breach of contract. Able

will be liable for breach of contract.

Beatrice purchased a rocking chair from the ABC Furniture Store. It was agreed that the store would deliver the chair to Beatrice's apartment. Beatrice lived in apartment number 4B. However, the chair was delivered to Beatrice's neighbor, Pamela, who lived in apartment number 4D. Pamela knew the rocking chair was meant for Beatrice, because Beatrice's name was on the invoice, but Pamela decided to keep the chair and not pay for it. If the ABC Furniture Store sued Pamela, it would:

win a lawsuit for quasi-contract and recover the reasonable value of the rocking chair.

Bill Cratchett leased an apartment from Grendel. Cratchett was a person of limited means in a locality where low-income housing was scarce. Shortly after signing the agreement, he fell in an unlit stairwell when a step unexpectedly gave way. In a suit for damages, Grendel relied on a clause in the lease stating, "Tenant agrees to hold Owner harmless from any claims for damages no matter how caused." Cratchett should

win because the exculpatory clause was unenforceable as a violation of public policy.

On July 1, Silk, Inc., sent Blue a telegram offering to sell Blue a building for $80,000. In the telegram, Silk stated that it would give Blue 30 days to accept the offer. On July 15, Blue sent Silk a telegram that included the following statement: "The price for your building seems too high. Would you consider taking $75,000?" This telegram was received by Silk on July 16. On July 19, Tint made an offer to Silk to purchase the building for $82,000. Upon learning of Tint's offer, Blue, on July 27, sent Silk a signed letter agreeing to purchase the building for $80,000. This letter was received by Silk on July 29. However, Silk now refuses to sell Blue the building. If Blue commences an action against Silk for breach of contract, Blue will

win, because Blue effectively accepted Silk's offer of July 1.

An auction at which the auctioneer may not withdraw an article put up for sale is said to be held:

without reserve

At an ABC University Alumni meeting, Carl and the others at his table signed a subscription form, on which he agreed to donate $5,000 to ABC University. In most states, Carl's promise:

would be enforced.


Related study sets

Understanding the value of money

View Set

Network Auth & Security Chapter 2

View Set

Chapter 08: Pain Lewis: Medical-Surgical Nursing, 10th Edition

View Set